Sunteți pe pagina 1din 15

PAPER INDUSTRIES CORPORATION OF THE PHILIPPINES Both the CTA and the Court of Appeals sustained the position

t of Appeals sustained the position of Picop and held


(PICOP), petitioner, that the interest deduction claimed by Picop was proper and allowable. In the
vs. instant Petition, the CIR insists on its original position.
COURT OF APPEALS, COMMISSIONER OF INTERNAL REVENUE
We begin by noting that interest payments on loans incurred by a taxpayer
and COURT OF TAX APPEALS, respondents.
(whether BOI-registered or not) are allowed by the NIRC as deductions
FACTS: against the taxpayer's gross income. Section 30 of the 1977 Tax Code
- The Paper Industries Corporation of the Philippines ("Picop") is a Philippine provided as follows:
corporation registered with the Board of Investments ("BOI") as a Sec. 30. Deduction from Gross Income. — The following may be
preferred pioneer enterprise with respect to its integrated pulp and paper deducted from gross income:
mill, and as a preferred non-pioneer enterprise with respect to its integrated
(a) Expenses:
plywood and veneer mills.
- On 21 April 1983, Picop received from the CIR two (2) letters of xxx xxx xxx
assessment and demand both dated 31 March 1983: (a) one for (b) Interest:
deficiency transaction tax and for documentary and science stamp tax;
and (b) the other for deficiency income tax for 1977, for an aggregate (1) In general. — The amount of interest paid within the taxable
amount of P88,763,255.00 (See full case for computation) year on indebtedness, except on indebtedness incurred or
continued to purchase or carry obligations the interest upon
- On 26 April 1983, Picop protested the assessment of deficiency which is exempt from taxation as income under this Title: . . .
transaction tax and documentary and science stamp taxes. Picop also
protested on 21 May 1983 the deficiency income tax assessment for 1977. Thus, the general rule is that interest expenses are deductible against gross
These protests were not formally acted upon by respondent CIR. On 26 income and this certainly includes interest paid under loans incurred in
September 1984, the CIR issued a warrant of distraint on personal property connection with the carrying on of the business of the taxpayer. 20
and a warrant of levy on real property against Picop, to enforce collection of In the instant case, the CIR does not dispute that the interest payments
the contested assessments; in effect, the CIR denied Picop's protests. were made by Picop on loans incurred in connection with the carrying on of
- Thereupon, Picop went before the CTA appealing the assessments. the registered operations of Picop, i.e., the financing of the purchase of
machinery and equipment actually used in the registered operations of
CTA- modified the findings of the CIR and holding Picop liable for the reduced Picop. Neither does the CIR deny that such interest payments were legally
aggregate amount of P20,133,762.33 (See case for computation) due and demandable under the terms of such loans, and in fact paid by
Picop during the tax year 1977.
Picop and the CIR both went to the Supreme Court on separate Petitions for
Review of the above decision of the CTA. In two (2) Resolutions, the Court The CIR has been unable to point to any provision of the 1977 Tax Code or any
referred the two (2) Petitions to the Court of Appeals. The Court of Appeals other Statute that requires the disallowance of the interest payments made by
consolidated the two (2) cases and rendered a decision, dated 31 August 1992, Picop. The CIR invokes Section 79 of Revenue Regulations No. 2 as amended
which further reduced the liability of Picop to P6,338,354.70. which reads as follows:
The dispositive portion of the Court of Appeals decision reads as follows: Sec. 79. Interest on Capital. — Interest calculated for cost-keeping or
other purposes on account of capital or surplus invested in the
WHEREFORE, the appeal of the Commissioner of Internal Revenue is
business, which does not represent a charge arising under an interest-
denied for lack of merit. The judgment against PICOP is modified, as
bearing obligation, is not allowable deduction from gross income.
follows:
We read the above provision of Revenue Regulations No. 2 as referring to
1. PICOP is declared liable for the 35% transaction tax in the amount of
so called "theoretical interest," that is to say, interest "calculated" or
P3,578,543.51;
computed (and not incurred or paid) for the purpose of determining the
2. PICOP is absolved from the payment of documentary and science stamp "opportunity cost" of investing funds in a given business. Such "theoretical"
tax of P300,000.00 and the compromise penalty of P300.00; or imputed interest does not arise from a legally demandable interest-bearing
obligation incurred by the taxpayer who however wishes to find out, e.g.,
3. PICOP shall pay 20% interest per annum on the deficiency income tax whether he would have been better off by lending out his funds and earning
of P1,481,579.15, for a period of three (3) years from 21 May 1983, or in interest rather than investing such funds in his business. One thing that Section
the total amount of P888,947.49, and a surcharge of 10% on the latter 79 quoted above makes clear is that interest which does constitute a charge
amount, or P88,984.75. arising under an interest-bearing obligation is an allowable deduction from
Picop and the CIR once more filed separate Petitions for Review before the gross income.
Supreme Court. It is claimed by the CIR that Section 79 of Revenue Regulations No. 2 was
PICOP’S CONTENTION: "patterned after" paragraph 1.266-1 (b), entitled "Taxes and Carrying Charges
- It is not liable at all to pay any of the assessments or any part thereof. Chargeable to Capital Account and Treated as Capital Items" of the U.S.
- It assails the propriety of the 35% deficiency transaction tax which the CTA Income Tax Regulations, which paragraph reads as follows:
held due from it in the amount of P3,578,543.51. (B) Taxes and Carrying Charges. — The items thus chargeable to
- Picop also questions the imposition by the Court of Appeals of the deficiency capital accounts are —
income tax of P1,481,579.15, resulting from disallowance of certain claimed
financial guarantee expenses and claimed year-end adjustments of sales and (11) In the case of real property, whether improved or unimproved and
cost of sales figures by Picop's external auditors. 3 whether productive or nonproductive.
CIR’S ARGUMENT: (a) Interest on a loan (but not theoretical interest of a taxpayer using
The CIR, upon the other hand, insists that the Court of Appeals erred in finding his own funds). 21
Picop not liable for surcharge and interest on unpaid transaction tax and for
The truncated excerpt of the U.S. Income Tax Regulations quoted by the CIR
documentary and science stamp taxes and in allowing Picop to claim as
needs to be related to the relevant provisions of the U.S. Internal Revenue
deductible expenses:
Code, which provisions deal with the general topic of adjusted basis for
(a) the net operating losses of another corporation (i.e., Rustan Pulp and determining allowable gain or loss on sales or exchanges of property and
Paper Mills, Inc.); and allowable depreciation and depletion of capital assets of the taxpayer:
(b) interest payments on loans for the purchase of machinery and Present Rule. The Internal Revenue Code, and the Regulations
equipment. promulgated thereunder provide that "No deduction shall be allowed
for amounts paid or accrued for such taxes and carrying charges as,
The CIR also claims that Picop should be held liable for interest at fourteen under regulations prescribed by the Secretary or his delegate, are
percent (14%) per annum from 15 April 1978 for three (3) years, and interest at chargeable to capital account with respect to property, if the taxpayer
twenty percent (20%) per annum for a maximum of three (3) years; and for a elects, in accordance with such regulations, to treat such taxes
surcharge of ten percent (10%), on Picop's deficiency income tax. Finally, the or charges as so chargeable."
CIR contends that Picop is liable for the corporate development tax equivalent
to five percent (5%) of its correct 1977 net income. At the same time, under the adjustment of basis provisions which have
just been discussed, it is provided that adjustment shall be made for all
COURT: "expenditures, receipts, losses, or other items" properly chargeable to
ISSUE (1): Whether Picop is entitled to deduct against current income a capital account, thus including taxes and carrying charges;
interest payments on loans for the purchase of machinery and equipment. however, an exception exists, in which event such adjustment to the
capital account is not made, with respect to taxes and carrying
YES. charges which the taxpayer has not elected to capitalize but for which
In 1969, 1972 and 1977, Picop obtained loans from foreign creditors in order to a deduction instead has been taken. 22 (Emphasis supplied)
finance the purchase of machinery and equipment needed for its operations. In The "carrying charges" which may be capitalized under the above quoted
its 1977 Income Tax Return, Picop claimed interest payments made in 1977, provisions of the U.S. Internal Revenue Code include, as the CIR has pointed
amounting to P42,840,131.00, on these loans as a deduction from its 1977 out, interest on a loan "(but not theoretical interest of a taxpayer using his
gross income. own funds)." What the CIR failed to point out is that such "carrying
The CIR disallowed this deduction upon the ground that, because the loans charges" may, at the election of the taxpayer, either be (a) capitalized in
had been incurred for the purchase of machinery and equipment, the interest which case the cost basis of the capital assets, e.g., machinery and
payments on those loans should have been capitalized instead and claimed equipment, will be adjusted by adding the amount of such interest
as a depreciation deduction taking into account the adjusted basis of the payments or alternatively, be (b) deducted from gross income of the taxpayer.
machinery and equipment (original acquisition cost plus interest charges) Should the taxpayer elect to deduct the interest payments against its gross
over the useful life of such assets. income, the taxpayer cannot at the same time capitalize the interest
payments. In other words, the taxpayer is not entitled to both the deduction The CIR disallowed all the deductions claimed on the basis of RPPM's losses,
from gross income and the adjusted (increased) basis for determining gain or apparently on two (2) grounds:
loss and the allowable depreciation charge. The U.S. Internal Revenue - Firstly, the previous losses were incurred by "another taxpayer," RPPM,
Code does not prohibit the deduction of interest on a loan obtained for
and not by Picop in connection with Picop's own registered operations. The
purchasing machinery and equipment against gross income, unless the
CIR took the view that Picop, RPPM and RMC were merged into one
taxpayer has also or previously capitalized the same interest payments and
(1) corporate personality only on 12 January 1978, upon approval of
thereby adjusted the cost basis of such assets.
the merger agreement by the BOI. Thus, during the taxable year 1977,
We have already noted that our 1977 NIRC does not prohibit the deduction of Picop on the one hand and RPPM and RMC on the other, still had their
interest on a loan incurred for acquiring machinery and equipment. Neither separate juridical personalities.
does our 1977 NIRC compel the capitalization of interest payments on such a - Secondly, the CIR alleged that these losses had been incurred by RPPM
loan. The 1977 Tax Code is simply silent on a taxpayer's right to elect one or
"from the borrowing of funds" and not from carrying out of RPPM's
the other tax treatment of such interest payments. Accordingly, the general rule
registered operations.
that interest payments on a legally demandable loan are deductible from gross
income must be applied. We focus on the first ground.
The CIR argues finally that to allow Picop to deduct its interest payments CTA: upheld the deduction claimed by Picop; its reasoning, however, is less
against its gross income would be to encourage fraudulent claims to double than crystal clear, especially in respect of its view of what the U.S. tax law was
deductions from gross income: on this matter. In any event, the CTA apparently fell back on the BOI opinion.
[t]o allow a deduction of incidental expense/cost incurred in the purchase of Respondent further averred that the incentives granted under Section 7 of
fixed asset in the year it was incurred would invite tax R.A. No. 5186 shall be available only to the extent in which they are
evasion through fraudulent application of double deductions from gross engaged in registered operations, citing Section 1 of Rule IX of the Basic
income. Rules and Regulations to Implement the Intent and Provisions of the
Investment Incentives Act, R.A. No. 5186.
The Court is not persuaded. So far as the records of the instant cases show,
Picop has not claimed to be entitled to double deduction of its 1977 interest We disagree with respondent. The purpose of the merger was to rationalize
payments. The CIR has neither alleged nor proved that Picop had previously the container board industry and not to take advantage of the net losses
adjusted its cost basis for the machinery and equipment purchased with the loan incurred by RPPMI prior to the stock swap. Thus, when stock of a
proceeds by capitalizing the interest payments here involved. The Court will corporation is purchased in order to take advantage of the corporation's net
not assume that the CIR would be unable or unwilling to disallow "a double operating loss incurred in years prior to the purchase, the corporation
deduction" should Picop, having deducted its interest cost from its gross thereafter entering into a trade or business different from that in which it
income, also attempt subsequently to adjust upward the cost basis of the was previously engaged, the net operating loss carry-over may be entirely
machinery and equipment purchased and claim, e.g., increased deductions for lost. Furthermore, once the BOI approved the merger agreement, the
depreciation. registered capacity of Rustan shall be transferred to PICOP, and the
previous losses of Rustan may be carried over by PICOP by operation
ISSUE (2): Whether Picop is entitled to deduct against current income net
of law. It is clear therefrom, that the deduction availed of under Section
operating losses incurred by Rustan Pulp and Paper Mills, Inc.
7(c) of R.A. No. 5186 was only proper."
NO.
In respect of the above underscored portion of the CTA decision, we must note
- On 18 January 1977, Picop entered into a merger agreement with the that the CTA in fact overlooked the statement made by petitioner's counsel
Rustan Pulp and Paper Mills, Inc. ("RPPM") and Rustan Manufacturing before the CTA that:
Corporation ("RMC"). Under this agreement, the rights, properties,
Among the attractions of the merger to Picop was the
privileges, powers and franchises of RPPM and RMC were to be
accumulated net operating loss carry-over of RMC that it
transferred, assigned and conveyed to Picop as the surviving corporation.
might possibly use to relieve it (Picop) from its income
The merger agreement was approved in 1977 by the creditors and stockholders
taxes, under Section 7 (c) of R.A. 5186. Said section
of Picop, RPPM and RMC and by the Securities and Exchange Commission.
provides:
Thereupon, on 30 November 1977, apparently the effective date of merger,
RPPM and RMC were dissolved. The Board of Investments approved the xxx xxx xxx
merger agreement on 12 January 1978.
With this benefit in mind, Picop addressed three (3)
- It appears that RPPM and RMC were, like Picop, BOI-registered companies. questions to the BOI in a letter dated November 25,
Immediately before merger effective date, RPPM had over preceding years 1976. The BOI replied on February 21, 1977 directly
accumulated losses in the total amount of P81,159,904.00. In its 1977 answering the three (3) queries. 30 (Emphasis supplied)
Income Tax Return, Picop claimed P44,196,106.00 of RPPM's accumulated
losses as a deduction against Picop's 1977 gross income. 24 The size of RPPM's accumulated losses as of the date of the merger
— more than P81,000,000.00 — must have constituted a powerful
- Upon the other hand, even before the effective date of merger, on 30 August attraction indeed for Picop.
1977, Picop sold all the outstanding shares of RMC stock to San Miguel
Corporation for the sum of P38,900,000.00, and reported a gain of The Court of Appeals followed the result reached by the CTA. The Court of
P9,294,849.00 from this transaction. Appeals, much like the CTA, concluded that since RPPM was dissolved on 30
November 1977, its accumulated losses were appropriately carried over by
In claiming such deduction, Picop relies on section 7 (c) of R.A. No. 5186 Picop in the latter's 1977 Income Tax Return "because by that time RPPMI and
which provides as follows: Picop were no longer separate and different taxpayers." 31
Sec. 7. Incentives to Registered Enterprise. — A registered enterprise, to The Court is unable to agree with the CTA and Court of Appeals on the
the extent engaged in a preferred area of investment, shall be granted the deductibility of RPPM's accumulated losses against Picop's 1977 gross
following incentive benefits: income.
(c) Net Operating Loss Carry-over. — A net operating loss incurred in any It is important to note at the outset that in our jurisdiction, the ordinary rule —
of the first ten years of operations may be carried over as a deduction from that is, the rule applicable in respect of corporations not registered with the BOI
taxable income for the six years immediately following the year of such as a preferred pioneer enterprise — is that net operating losses cannot be
loss. The entire amount of the loss shall be carried over to the first of the carried over. Under our Tax Code, both in 1977 and at present, losses may
six taxable years following the loss, and any portion of such loss which be deducted from gross income only if such losses were actually sustained
exceeds the taxable income of such first year shall be deducted in like in the same year that they are deducted or charged off. Section 30 of the
manner from the taxable income of the next remaining five years. The net 1977 Tax Code provides:
operating loss shall be computed in accordance with the provisions of the
National Internal Revenue Code, any provision of this Act to the contrary Sec. 30. Deductions from Gross Income. — In computing net income,
notwithstanding, except that income not taxable either in whole or in part there shall be allowed as deduction —
under this or other laws shall be included in gross income. (d) Losses:
Picop had secured a letter-opinion from the BOI dated 21 February 1977 — that (1) By Individuals. — In the case of an individual, losses actually
is, after the date of the agreement of merger but before the merger became sustained during the taxable year and not compensated for by an
effective — relating to the deductibility of the previous losses of RPPM under insurance or otherwise —
Section 7 (c) of R.A. No. 5186 as amended. The pertinent portions of this BOI
opinion, signed by BOI Governor Cesar Lanuza, read as follows: (A) If incurred in trade or business;

2) PICOP will not be allowed to carry over the losses of Rustan prior to xxx xxx xxx
the legal dissolution of the latter because at that time the two (2) (2) By Corporations. — In a case of a corporation, all losses actually
companies still had separate legal personalities; sustained and charged off within the taxable year and not
3) After BOI approval of the merger, PICOP can no longer apply for the compensated for by insurance or otherwise.
registration of the registered capacity of Rustan because with the approved (3) By Non-resident Aliens or Foreign Corporations. — In the case of
merger, such registered capacity of Rustan transferred to PICOP will have a non-resident alien individual or a foreign corporation, the losses
the same registration date as that of Rustan. In this case, the previous deductible are those actually sustained during the year incurred in
losses of Rustan may be carried over by PICOP, because with the merger, business or trade conducted within the Philippines, . . . 32
PICOP assumes all the rights and obligations of Rustan subject, however,
to the period prescribed for carrying over of such losses. Section 76 of the Philippine Income Tax Regulations (Revenue Regulation No.
2, as amended) is even more explicit and detailed:
Curiously enough, Picop did not also seek a ruling on this matter, clearly a
matter of tax law, from the Bureau of Internal Revenue. Picop chose to rely Sec. 76. When charges are deductible. — Each year's return, so far as
solely on the BOI letter-opinion. practicable, both as to gross income and deductions therefrom should
be complete in itself, and taxpayers are expected to make every ISSUE (3): Whether Picop is entitled to deduct against current income
reasonable effort to ascertain the facts necessary to make a correct certain claimed financial guarantee expenses.
return. The expenses, liabilities, or deficit of one year cannot be used
In its Income Tax Return for 1977, Picop also claimed a deduction in the
to reduce the income of a subsequent year. A taxpayer has the right to
amount of P1,237,421.00 as financial guarantee expenses.
deduct all authorized allowances and it follows that if he does not
within any year deduct certain of his expenses, losses, interests, taxes, This deduction is said to relate to chattel and real estate mortgages required
or other charges, he can not deduct them from the income of the next from Picop by the PNB and DBP as guarantors of loans incurred by Picop from
or any succeeding year. . . . foreign creditors. According to Picop, the claimed deduction represents
registration fees and other expenses incidental to registration of mortgages
xxx xxx xxx
in favor of DBP and PNB.
. . . . If subsequent to its occurrence, however, a taxpayer first
In support of this claimed deduction, Picop allegedly showed its own
ascertains the amount of a loss sustained during a prior taxable
vouchers to BIR Examiners to prove disbursements to the Register of
year which has not been deducted from gross income, he may
Deeds of Tandag, Surigao del Sur, of particular amounts. In the
render an amended return for such preceding taxable year including
proceedings before the CTA, however, Picop did not submit in evidence
such amount of loss in the deduction from gross income and may in
such vouchers and instead presented one of its employees to testify that the
proper cases file a claim for refund of the excess paid by reason of the
amount claimed had been disbursed for the registration of chattel and real
failure to deduct such loss in the original return. A loss from theft or
estate mortgages.
embezzlement occurring in one year and discovered in another is
ordinarily deductible for the year in which sustained. The CIR disallowed this claimed deduction upon the ground of
insufficiency of evidence. This disallowance was sustained by the CTA and the
It is thus clear that under our law, and outside the special realm of BOI-
Court of Appeals. The CTA said:
registered enterprises, there is no such thing as a carry-over of net operating
loss. To the contrary, losses must be deducted against current income in the No records are available to support the abovementioned expenses. The
taxable year when such losses were incurred. Moreover, such losses may be vouchers merely showed that the amounts were paid to the Register of
charged off only against income earned in the same taxable year when the Deeds and simply cash account. Without the supporting papers such
losses were incurred. as the invoices or official receipts of the Register of Deeds, these
vouchers standing alone cannot prove that the payments made were
Thus it is that R.A. No. 5186 introduced the carry-over of net operating
for the accrued expenses in question. The best evidence of payment is
losses as a very special incentive to be granted only to registered pioneer
the official receipts issued by the Register of Deeds. The testimony of
enterprises and only with respect to their registered operations. The
petitioner's witness that the official receipts and cash vouchers were
statutory purpose here may be seen to be the encouragement of the
shown to the Bureau of Internal Revenue will not suffice if no records
establishment and continued operation of pioneer industries by allowing
could be presented in court for proper marking and identification.
the registered enterprise to accumulate its operating losses which may be
expected during the early years of the enterprise and to permit the The Court of Appeals added:
enterprise to offset such losses against income earned by it in later years
after successful establishment and regular operations. To promote its The mere testimony of a witness for PICOP and the cash vouchers do
economic development goals, the Republic foregoes or defers taxing the not suffice to establish its claim that registration fees were paid to the
income of the pioneer enterprise until after that enterprise has recovered or Register of Deeds for the registration of real estate and chattel
offset its earlier losses. We consider that the statutory purpose can be served mortgages in favor of Development Bank of the Philippines and the
only if the accumulated operating losses are carried over and charged off Philippine National Bank as guarantors of PICOP's loans. The witness
against income subsequently earned and accumulated by the same could very well have been merely repeating what he was instructed to
enterprise engaged in the same registered operations. say regardless of the truth, while the cash vouchers, which we do not
find on file, are not said to provide the necessary details regarding the
In the instant case, to allow the deduction claimed by Picop would be to nature and purpose of the expenses reflected therein. PICOP should
permit one corporation or enterprise, Picop, to benefit from the operating have presented, through the guarantors, its owner's copy of the
losses accumulated by another corporation or enterprise, RPPM. RPPM far registered titles with the lien inscribed thereon as well as an official
from benefiting from the tax incentive granted by the BOI statute, in fact gave receipt from the Register of Deeds evidencing payment of the
up the struggle and went out of existence and its former stockholders joined the registration fee.
much larger group of Picop's stockholders. To grant Picop's claimed deduction
would be to permit Picop to shelter its otherwise taxable income (an objective We must support the CTA and the Court of Appeals in their foregoing
which Picop had from the very beginning) which had not been earned by the rulings. A taxpayer has the burden of proving entitlement to a claimed
registered enterprise which had suffered the accumulated losses. In effect, to deduction. In the instant case, even Picop's own vouchers were not submitted
grant Picop's claimed deduction would be to permit Picop to purchase a in evidence and the BIR Examiners denied that such vouchers and other
tax deduction and RPPM to peddle its accumulated operating losses. Under documents had been exhibited to them. Moreover, cash vouchers can only
the CTA and Court of Appeals decisions, Picop would benefit by immunizing confirm the fact of disbursement but not necessarily the purpose thereof. 37 The
P44,196,106.00 of its income from taxation thereof although Picop had not run best evidence that Picop should have presented to support its claimed
the risks and incurred the losses which had been encountered and suffered by deduction were the invoices and official receipts issued by the Register of
RPPM. Conversely, the income that would be shielded from taxation is not Deeds. Picop not only failed to present such documents; it also failed to explain
income that was, after much effort, eventually generated by the same the loss thereof, assuming they had existed before. 38 Under the best evidence
registered operations which earlier had sustained losses. We consider and so rule, 39 therefore, the testimony of Picop's employee was inadmissible and was
hold that there is nothing in Section 7 (c) of R.A. No. 5186 which either in any case entitled to very little, if any, credence.
requires or permits such a result. Indeed, that result makes non-sense of the We consider that entitlement to Picop's claimed deduction of P1,237,421.00
legislative purpose which may be seen clearly to be projected by Section 7 (c), was not adequately shown and that such deduction must be disallowed.
R.A. No. 5186.
The CTA and the Court of Appeals allowed the offsetting of RPPM's
accumulated operating losses against Picop's 1977 gross income, basically
because towards the end of the taxable year 1977, upon the arrival of the
effective date of merger, only one (1) corporation, Picop, remained. The losses
suffered by RPPM's registered operations and the gross income generated by
Picop's own registered operations now came under one and the same corporate
roof.
- We consider that this circumstance relates much more to form than to
substance. We do not believe that that single purely technical factor is
enough to authorize and justify the deduction claimed by Picop. Picop's
claim for deduction is not only bereft of statutory basis; it does violence to the
legislative intent which animates the tax incentive granted by Section 7 (c) of
R.A. No. 5186. In granting the extraordinary privilege and incentive of a
net operating loss carry-over to BOI-registered pioneer enterprises, the
legislature could not have intended to require the Republic to forego tax
revenues in order to benefit a corporation which had run no risks and
suffered no losses, but had merely purchased another's losses.
Both the CTA and the Court of Appeals appeared much impressed not only with
corporate technicalities but also with the U.S. tax law on this matter. It should
suffice, however, simply to note that in U.S. tax law, the availability to
companies generally of operating loss carry-overs and of operating loss carry-
backs is expressly provided and regulated in great detail by statute. 33 In our
jurisdiction, save for Section 7 (c) of R.A. No. 5186, no statute recognizes or
permits loss carry-overs and loss carry-backs. Indeed, as already noted, our tax
law expressly rejects the very notion of loss carry-overs and carry-backs.
We conclude that the deduction claimed by Picop in the amount of
P44,196,106.00 in its 1977 Income Tax Return must be disallowed.
ESSO STANDARD EASTERN, INC., (formerly, Standard-Vacuum Oil In Caltex (Phil.) Inc. v. Acting Commissioner of Customs, 2 the Court stated
Company), petitioner, through Justice Jose P. Bengzon:
vs.
A margin levy on foreign exchange is a form of exchange control or
THE COMMISSIONER OF INTERNAL REVENUE, respondent.
restriction designed to discourage imports and encourage exports, and
FACTS: In CTA Case No. 1251, petitioner ESSO deducted from its gross ultimately, 'curtail any excessive demand upon the international reserve' in
income for 1959, as part of its ordinary and necessary business expenses, order to stabilize the currency. Originally adopted to cope with balance of
the amount it had spent for drilling and exploration of its petroleum payment pressures, exchange restrictions have come to serve various purposes,
concessions. such as limiting non-essential imports, protecting domestic industry and when
combined with the use of multiple currency rates providing a source of revenue
CIR: disallowed the claim on the ground that the expenses should be to the government, and are in many developing countries regarded as a more or
capitalized and might be written off as a loss only when a "dry hole" less inevitable concomitant of their economic development programs. The
should result. different measures of exchange control or restriction cover different phases of
ESSO then filed an amended return where it asked for the refund of foreign exchange transactions, i.e., in quantitative restriction, the control is on
P323,279.00 by reason of its abandonment as dry holes of several of its oil the amount of foreign exchange allowable. In the case of the margin levy, the
wells. Also claimed as ordinary and necessary expenses in the same return immediate impact is on the rate of foreign exchange; in fact, its main function
was the amount of P340,822.04, representing margin fees it had paid to the is to control the exchange rate without changing the par value of the peso as
Central Bank on its profit remittances to its New York head office. fixed in the Bretton Woods Agreement Act. For a member nation is not
supposed to alter its exchange rate (at par value) to correct a merely temporary
CIR: granted a tax credit of P221,033.00 only, disallowing the claimed disequilibrium in its balance of payments. By its nature, the margin levy is part
deduction for the margin fees paid. of the rate of exchange as fixed by the government.
In CTA Case No. 1558, the CIR assessed ESSO a deficiency income tax for As to the contention that the margin levy is a tax on the purchase of foreign
the year 1960, in the amount of P367,994.00, plus 18% interest thereon of exchange and hence should not form part of the exchange rate, suffice it to state
P66,238.92 for the period from April 18,1961 to April 18, 1964, for a total of that We have already held the contrary for the reason that a tax is levied to
P434,232.92. provide revenue for government operations, while the proceeds of the
The deficiency arose from the disallowance of the margin fees of margin fee are applied to strengthen our country's international reserves.
Pl,226,647.72 paid by ESSO to the Central Bank on its profit remittances to Earlier, in Chamber of Agriculture and Natural Resources of the Philippines
its New York head office. v. Central Bank, 3 the same idea was expressed, though in connection with a
ESSO settled this deficiency assessment on August 10, 1964, by applying the different levy, through Justice J.B.L. Reyes:
tax credit of P221,033.00 representing its overpayment on its income tax Neither do we find merit in the argument that the 20% retention of exporter's
for 1959 and paying under protest the additional amount of P213,201.92. foreign exchange constitutes an export tax. A tax is a levy for the purpose of
On August 13, 1964, it claimed the refund of P39,787.94 as overpayment on providing revenue for government operations, while the proceeds of the 20%
the interest on its deficiency income tax. retention, as we have seen, are applied to strengthen the Central Bank's
international reserve.
ESSO argued: that the 18% interest should have been imposed not on the
total deficiency of P367,944.00 but only on the amount of P146,961.00, the We conclude then that the margin fee was imposed by the State in the
difference between the total deficiency and its tax credit of P221,033.00. exercise of its police power and not the power of taxation.

CIR: denied the claim, who insisted on charging the 18% interest on the entire ESSO prays: that if margin fees are not taxes, they should nevertheless be
amount of the deficiency tax. considered necessary and ordinary business expenses and therefore still
deductible from its gross income.
- also denied the claims of ESSO for refund of the overpayment of
its 1959 and 1960 income taxes, holding that the margin fees paid COURT: The fees were paid for the remittance by ESSO as part of the profits
to the Central Bank could not be considered taxes or allowed as to the head office in the Unites States. Such remittance was an expenditure
deductible business expenses. necessary and proper for the conduct of its corporate affairs.

ESSO appealed to the CTA and sought the refund of P102,246.00 for 1959. The applicable provision is Section 30(a) of the National Internal Revenue
Code reading as follows:
ESSO CONTENTION: that the margin fees were deductible from gross
income either as a tax or as an ordinary and necessary business expense. SEC. 30. Deductions from gross income in computing net income there shall be
allowed as deductions
- It also claimed an overpayment of its tax by P434,232.92 in 1960,
for the same reason. (a) Expenses:
- Additionally, ESSO argued that even if the amount paid as margin (1) In general. — All the ordinary and necessary expenses paid or incurred
fees were not legally deductible, there was still an overpayment during the taxable year in carrying on any trade or business, including a
by P39,787.94 for 1960, representing excess interest. reasonable allowance for salaries or other compensation for personal services
actually rendered; traveling expenses while away from home in the pursuit of a
CTA: denied petitioner's claim for refund of P102,246.00 for 1959 and trade or business; and rentals or other payments required to be made as a
P434,234.92 for 1960 but sustained its claim for P39,787.94 as excess condition to the continued use or possession, for the purpose of the trade or
interest. business, of property to which the taxpayer has not taken or is not taking title or
ISSUE: WON R.A. 2009, entitled An Act to Authorize the Central Bank of in which he has no equity.
the Philippines to Establish a Margin Over Banks' Selling Rates of Foreign (2) Expenses allowable to non-resident alien individuals and foreign
Exchange, is a police measure or a revenue measure. POLICE MEASURE! corporations. — In the case of a non-resident alien individual or a foreign
WON CTA erred in denying the claim of refund of petitioner. NO! corporation, the expenses deductible are the necessary expenses paid or
incurred in carrying on any business or trade conducted within the Philippines
TAKE NOTE: If it is a revenue measure, the margin fees paid by the exclusively.
petitioner to the Central Bank on its profit remittances to its New York head
office should be deductible from ESSO's gross income under Sec. 30(c) of In the case of Atlas Consolidated Mining and Development Corporation v.
the National Internal Revenue Code. This provides that all taxes paid or Commissioner of Internal Revenue, 4 the Court laid down the rules on the
accrued during or within the taxable year and which are related to the deductibility of business expenses, thus:
taxpayer's trade, business or profession are deductible from gross income. The principle is recognized that when a taxpayer claims a deduction, he must
PETITIONER MAINTAINS: that margin fees are taxes and cites the point to some specific provision of the statute in which that deduction is
background and legislative history of the Margin Fee Law showing that R.A. authorized and must be able to prove that he is entitled to the deduction
2609 was nothing less than a revival of the 17% excise tax on foreign exchange which the law allows. As previously adverted to, the law allowing expenses as
imposed by R.A. 601. deduction from gross income for purposes of the income tax is Section 30(a)
(1) of the National Internal Revenue which allows a deduction of 'all the
- This was a revenue measure formally proposed by President ordinary and necessary expenses paid or incurred during the taxable year in
Carlos P. Garcia to Congress as part of, and in order to balance, the carrying on any trade or business.' An item of expenditure, in order to be
budget for 1959-1960. It was enacted by Congress as such and, deductible under this section of the statute, must fall squarely within its
significantly, properly originated in the House of Representatives. language.
During its two and a half years of existence, the measure was one of
the major sources of revenue used to finance the ordinary operating We come, then, to the statutory test of deductibility where it is axiomatic that
expenditures of the government. It was, moreover, payable out of to be deductible as a business expense, three conditions are imposed,
the General Fund. namely:
(1) the expense must be ordinary and necessary,
CTA: well-settled jurisprudence that only in extremely doubtful matters of (2) it must be paid or incurred within the taxable year, and
interpretation does the legislative history of an act of Congress become (3) it must be paid or incurred in carrying on a trade or business.
important. As a matter of fact, there may be no resort to the legislative history
of the enactment of a statute, the language of which is plain and unambiguous, In addition, not only must the taxpayer meet the business test, he must
since such legislative history may only be resorted to for the purpose of substantially prove by evidence or records the deductions claimed under
solving doubt, not for the purpose of creating it. [50 Am. Jur. 328.] the law, otherwise, the same will be disallowed. The mere allegation of the
taxpayer that an item of expense is ordinary and necessary does not justify
- Apart from the above consideration, there are at least two cases its deduction.
where we have held that a margin fee is not a tax but an exaction
designed to curb the excessive demands upon our international While it is true that there is a number of decisions in the United States delving
reserve. on the interpretation of the terms 'ordinary and necessary' as used in the federal
tax laws, no adequate or satisfactory definition of those terms is possible.
Similarly, this Court has never attempted to define with precision the COMMISSIONER OF INTERNAL REVENUE, petitioner,
terms 'ordinary and necessary.' There are however, certain guiding principles vs.
worthy of serious consideration in the proper adjudication of conflicting claims. COURT OF TAX APPEALS and SMITH KLINE & FRENCH
OVERSEAS CO. (PHILIPPINE BRANCH), respondents.
Ordinarily, an expense will be considered 'necessary' where the expenditure
is appropriate and helpful in the development of the taxpayer's business. It FACTS:
is 'ordinary' when it connotes a payment which is normal in relation to the
- Smith Kline and French Overseas Company, a multinational firm domiciled in
business of the taxpayer and the surrounding circumstances.
Philadelphia, Pennsylvania, is licensed to do business in the Philippines. It is
The term 'ordinary' does not require that the payments be habitual or normal engaged in the importation, manufacture and sale of pharmaceuticals drugs and
in the sense that the same taxpayer will have to make them often; the payment chemicals.
may be unique or non-recurring to the particular taxpayer affected.
- In its 1971 original income tax return, Smith Kline declared a net taxable
There is thus no hard and fast rule on the matter. The right to a deduction income of P1,489,277 and paid P511,247 as tax due. Among the deductions
depends in each case on the particular facts and the relation of the claimed from gross income was P501,040 ($77,060) as its share of the head
payment to the type of business in which the taxpayer is engaged. office overhead expenses. However, in its amended return filed on March 1,
1973, there was an overpayment of P324,255 "arising from underdeduction
The intention of the taxpayer often may be the controlling fact in making the of home office overhead" (Exh. E). It made a formal claim for the refund
determination. Assuming that the expenditure is ordinary and necessary in of the alleged overpayment.
the operation of the taxpayer's business, the answer to the question as to
whether the expenditure is an allowable deduction as a business expense must - It appears that sometime in October, 1972, Smith Kline received from its
be determined from the nature of the expenditure itself, which in turn international independent auditors, Peat, Marwick, Mitchell and Company,
depends on the extent and permanency of the work accomplished by the an authenticated certification to the effect that the Philippine share in the
expenditure. unallocated overhead expenses of the main office for the year ended
December 31, 1971 was actually $219,547 (P1,427,484). It further stated in
In the light of the above explanation, we hold that the Court of Tax Appeals did the certification that the allocation was made on the basis of the percentage of
not err when it held on this issue as follows: gross income in the Philippines to gross income of the corporation as a whole.
Considering the foregoing test of what constitutes an ordinary and necessary By reason of the new adjustment, Smith Kline's tax liability was greatly
deductible expense, it may be asked: Were the margin fees paid by petitioner on reduced from P511,247 to P186,992 resulting in an overpayment
its profit remittance to its Head Office in New York appropriate and helpful in of P324,255.
the taxpayer's business in the Philippines? Were the margin fees incurred for CTA: ordered the Commissioner to refund the overpayment or grant a tax
purposes proper to the conduct of the affairs of petitioner's branch in the credit to Smith Kline. The Commissioner appealed to this Court.
Philippines? Or were the margin fees incurred for the purpose of realizing a
profit or of minimizing a loss in the Philippines? Obviously not. ISSUE:
As stated in the Lopez case, the margin fees are not expenses in connection WON Smith Kline's amended 1971 return is in conformity with the law and
with the production or earning of petitioner's incomes in the Philippines. regulations?
They were expenses incurred in the disposition of said incomes; expenses
COURT: YES
for the remittance of funds after they have already been earned by petitioner's
branch in the Philippines for the disposal of its Head Office in New York which The governing law is found in section 37 of the old National Internal Revenue
is already another distinct and separate income taxpayer. Code, Commonwealth Act No. 466, which is reproduced in Presidential Decree
No. 1158, the National Internal Revenue Code of 1977 and which reads:
xxx
SEC. 37. Income form sources within the Philippines. —
Since the margin fees in question were incurred for the remittance of funds to
petitioner's Head Office in New York, which is a separate and distinct income (b) Net income from sources in the Philippines. — From the items of
taxpayer from the branch in the Philippines, for its disposal abroad, it can gross income specified in subsection (a) of this section there shall be
never be said therefore that the margin fees were appropriate and helpful deducted the expenses, losses, and other deductions properly
in the development of petitioner's business in the Philippines exclusively or apportioned or allocated thereto and a ratable part of any expenses,
were incurred for purposes proper to the conduct of the affairs of losses, or other deductions which cannot definitely be allocated to
petitioner's branch in the Philippines exclusively or for the purpose of some item or class of gross income. The remainder, if any, shall be
realizing a profit or of minimizing a loss in the Philippines exclusively. included in full as net income from sources within the Philippines.
If at all, the margin fees were incurred for purposes proper to the conduct of Revenue Regulations No. 2 of the Department of Finance contains the
the corporate affairs of Standard Vacuum Oil Company in New York, but following provisions on the deductions to be made to determine the net income
certainly not in the Philippines. from Philippine sources:
ESSO has not shown that the remittance to the head office of part of its SEC. 160. Apportionment of deductions. — From the items specified
profits was made in furtherance of its own trade or business. in section 37(a), as being derived specifically from sources within the
Philippines there shall be deducted the expenses, losses, and other
The petitioner merely presumed that all corporate expenses are necessary and
deductions properly apportioned or allocated thereto and a ratable part
appropriate in the absence of a showing that they are illegal or ultra vires. This
of any other expenses, losses or deductions which can not definitely
is error. The public respondent is correct when it asserts that "the paramount
be allocated to some item or class of gross income. The remainder
rule is that claims for deductions are a matter of legislative grace and do not
shall be included in full as net income from sources within the
turn on mere equitable considerations ... . The taxpayer in every instance has
Philippines. The ratable part is based upon the ratio of gross income
the burden of justifying the allowance of any deduction claimed." 5
from sources within the Philippines to the total gross income.
It is clear that ESSO, having assumed an expense properly attributable to
Example: A non-resident alien individual whose taxable year is the
its head office, cannot now claim this as an ordinary and necessary expense
calendar year, derived gross income from all sources for 1939 of
paid or incurred in carrying on its own trade or business.
P180,000, including therein:
Interest on bonds of a domestic corporation P9,000
Dividends on stock of a domestic corporation 4,000
Royalty for the use of patents within the Philippines 12,000
Gain from sale of real property located within the Philippines 11,000
Total P36,000
that is, one-fifth of the total gross income was from sources within the
Philippines. The remainder of the gross income was from sources
without the Philippines, determined under section 37(c).
The expenses of the taxpayer for the year amounted to P78,000. Of
these expenses the amount of P8,000 is properly allocated to income
from sources within the Philippines and the amount of P40,000 is
properly allocated to income from sources without the Philippines.
The remainder of the expense, P30,000, cannot be definitely allocated
to any class of income. A ratable part thereof, based upon the relation
of gross income from sources within the Philippines to the total gross
income, shall be deducted in computing net income from sources
within the Philippines. Thus, these are deducted from the P36,000 of
gross income from sources within the Philippines expenses amounting
to P14,000 [representing P8,000 properly apportioned to the income
from sources within the Philippines and P6,000, a ratable part (one-
fifth) of the expenses which could not be allocated to any item or class
of gross income.] The remainder, P22,000, is the net income from
sources within the Philippines.
From the foregoing provisions, it is manifest that where an expense is clearly
related to the production of Philippine-derived income or to Philippine
operations (e.g. salaries of Philippine personnel, rental of office building in the COMMISSIONER OF INTERNAL REVENUE, petitioner,
Philippines), that expense can be deducted from the gross income acquired vs.
in the Philippines without resorting to apportionment. CARLOS PALANCA, JR., respondent.
The overhead expenses incurred by the parent company in connection with FACTS: Sometime in July, 1950, the late Don Carlos Palanca, Sr. donated in
finance, administration, and research and development, all of which direct favor of his son, the petitioner, herein shares of stock in La Tondeña, Inc.
benefit its branches all over the world, including the Philippines, fall under amounting to 12,500 shares.
a different category however. These are items which cannot be definitely
For failure to file a return on the donation within the statutory period, the
allocated or Identified with the operations of the Philippine branch. For
petitioner was assessed the sums of P97,691.23, P24,442.81 and P47,868.70
1971, the parent company of Smith Kline spent $1,077,739. Under section
as gift tax, 25% surcharge and interest, respectively, which he paid on June
37(b) of the Revenue Code and section 160 of the regulations, Smith Kline
22, 1955.
can claim as its deductible share a ratable part of such expenses based upon
the ratio of the local branch's gross income to the total gross income, On March 1, 1956, the petitioner filed with the Bureau of Internal Revenue
worldwide, of the multinational corporation. his income tax return for the calendar year 1955, claiming, among others, a
deduction for interest amounting to P9,706.45 and reporting a taxable
CIR’S CONTENTION:
income of P65,982.12.
- In his petition for review, the Commissioner does not dispute the right of
Smith Kline to avail itself of section 37(b) of the Tax Code and section 160 of
the regulations. But the Commissioner maintains that such right is not Taxes withheld by La Tondeña Inc. from Mr. Palanca's wages P13,172.41
absolute and that as there exists a contract (in this case a service
Payment under Income Tax Receipt No. 677395 dated May 11,
agreement) which Smith Kline has entered into with its home office,
1956 3,939.80
prescribing the amount that a branch can deduct as its share of the main
office's overhead expenses, that contract is binding. Payment under Income Tax Receipt dated August 14, 1956 3,939.80
- The Commissioner contends that since the share of the Philippine branch
has been fixed at $77,060, Smith Kline itself cannot claim more than the P21,052.01
said amount. To allow Smith Kline to deduct more than what was expressly On the basis of this return, he was assessed the sum of P21,052.91, as income
provided in the agreement would be to ignore its existence. It is a cardinal rule tax, which he paid, as follows:
that a contract is the law between the contracting parties and the stipulations
therein must be respected unless these are proved to be contrary to law, morals, Subsequently, on November 10, 1956, the petitioner filed an amended return
good customs and public policy. There being allegedly no showing to the for the calendar year 1955, claiming therein an additional deduction in the
contrary, the provisions thereof must be followed. amount of P47,868.70 representing interest paid on the donee's gift tax,
thereby reporting a taxable net income of P18,113.42 and a tax due thereon in
- The Commissioner also argues that the Tax Court erred in relying on the the sum of P3,167.00.
certification of Peat, Marwick, Mitchell and Company that Smith Kline is
entitled to deduct P1,427,484 ($219,547) as its allotted share and that Smith PETITIONER BASIS: Section 30(b) (1) of the Tax Code, which authorizes
Kline has not presented any evidence to show that the home office expenses the deduction from gross income of interest paid within the taxable year on
chargeable to Philippine operations exceeded $77,060. indebtedness. A claim for the refund of alleged overpaid income taxes for the
year 1955 amounting to P17,885.01, which is the difference between the
SMITH KLINE’S ARGUMENT: amount of P21,052.01 he paid as income taxes under his original return and of
- the contract between itself and its home office cannot amend tax laws and P3,167.00, was filed together with this amended return.
regulations. The matter of allocated expenses which are deductible under the
law cannot be the subject of an agreement between private parties nor can the In a communication dated June 20, 1957, the respondent (BIR) denied the
Commissioner acquiesce in such an agreement. claim for refund.
- Smith Kline had to amend its return because it is of common knowledge that Meanwhile, the Bureau of Internal Revenue considered the transfer of
audited financial statements are generally completed three or four months after 12,500 shares of stock of La Tondeña Inc. to be a transfer in contemplation of
the close of the accounting period. There being no financial statements yet death pursuant to Section 88(b) of the National Internal Revenue Code
when the certification of January 11, 1972 was made the treasurer could not
have correctly computed Smith Kline's share in the home office overhead Consequently, the respondent assessed against the petitioner the sum of
expenses in accordance with the gross income formula prescribed in section P191,591.62 as estate and inheritance taxes on the transfer of said 12,500
160 of the Revenue Regulations. What the treasurer certified was a mere shares of stock.
estimate. The amount of P17,002.74 paid on June 22, 1955 by the petitioner as gift tax,
- Smith Kline likewise submits that it has presented ample evidence to support including interest and surcharge, under Official Receipt No. 2855 was applied
its claim for refund. To this end, it has presented before the Tax Court the to his estate and inheritance tax liability.
authenticated statement of Peat, Marwick, Mitchell and Company to show that
since the gross income of the Philippine branch was P7,143,155 ($1,098,617) 1% monthly interest on P76,724.38 P22,633.69
for 1971 as per audit report prepared by Sycip, Gorres, Velayo and Company, September 2, 1952 to February 16, 1955
and the gross income of the corporation as a whole was $6,891,052, Smith
Kline's share at 15.94% of the home office overhead expenses was P1,427,484 1% monthly interest on P71,264.77 1,068.97
($219,547) February 16, 1955 to March 31, 1955

Clearly, the weight of evidence bolsters its position that the amount of 1% monthly interest on P114,867.24 4,287.99
P1,427,484 represents the correct ratable share, the same having been September 2, 1952 to April 16, 1953
computed pursuant to section 37(b) and section 160.
1% monthly interest on P50,832.77 1,372.48
In a manifestation dated July 19, 1983, Smith Kline declared that with respect March 31, 1955 to June 22, 1955
to its share of the head office overhead expenses in its income tax returns for
the years 1973 to 1981, it deducted its ratable share of the total overhead 1% monthly interest on P119,155.23 31,218.67
expenses of its head office for those years as computed by the independent April 16, 1953 to June 22, 1955
auditors hired by the parent company in Philadelphia, Pennsylvania U.S.A., as
Total P60,581.80
soon as said computations were made available to it.
On the tax liability of P191,591.62, the petitioner paid the amount of
We hold that Smith Kline's amended 1971 return is in conformity with the P60,581.80 as interest for delinquency as follows:
law and regulations. The Tax Court correctly held that the refund or credit
of the resulting overpayment is in order. On August 12, 1958, the petitioner once more filed an amended income tax
return for the calendar year 1955, claiming, in addition to the interest deduction
of P9,076.45 appearing in his original return, a deduction in the amount of
P60,581.80, representing interest on the estate and inheritance taxes on the
12,500 shares of stock, thereby reporting a net taxable income for 1955 in the
amount of P5,400.32 and an income tax due thereon in the sum of P428.00.
Attached to this amended return was a letter of the petitioner, dated August 11,
1958, wherein he requested the refund of P20,624.01 which is the difference
between the amounts of P21,052.01 he paid as income tax under his original
return and of P428.00.
Without waiting for the respondent's decision on this claim for refund, the
petitioner filed his petition for review before this Court on August 13, 1958.
On July 24, 1959, the respondent denied the petitioner's request for the refund
of the sum of P20,624.01.
CTA: that the amount paid by respondent Palanca for interest on his delinquent
estate and inheritance tax is deductible from the gross income for that year
under Section 30 (b) (1) of the Revenue Code, and, that said respondent's claim
for refund therefor has not prescribed.
The Commissioner of Internal Revenue now seeks the reversal of the Court of
Tax Appeal's ruling on the aforementioned petition for review.
Commissioner urges: that a tax is not an indebtedness. Citing American Actually, this second assessment by the Bureau was for the same transaction as
cases, he argues that there is a material and fundamental distinction between a that for which they assessed respondent Palanca the above donee's gift tax. The
"tax" and a "debt.” Bureau, however, on further consideration, decided that the donation of the
stocks in question was made in contemplation of death, and hence, should
He adopts the view that "debts are due to the government in its corporate be assessed as an inheritance. Thus the second assessment.
capacity, while taxes are due to the government in its sovereign capacity. A
debt is a sum of money due upon contract express or implied or one which is The first claim was denied by the petitioner for the first time on June 20, 1957.
evidenced by a judgment. Taxes are imposts levied by government for its Thereafter, the said denial was twice reiterated, on October 14, 1957 and
support or some special purpose which the government has recognized." November 7, 1957, upon respondent Palanca's plea for the reconsideration of
the ruling of June 20, 1957.
In view of the distinction, then, the Commissioner submits that the
deductibility of "interest on indebtedness" from a person's income tax The second claim was filed with the Court of Tax Appeals on August 13, 1958,
under Section 30(b) (1) cannot extend to "interest on taxes." or even before the same had been denied by the petitioner. Respondent
Palanca's second claim was denied by the latter on July 24, 1959.
ISSUE: Whether the interest on the delinquent estate and inheritance tax
is deductible from the gross income PETITIONER CONTENTION: that under Section 11 of Republic Act
1124,1 the herein claimant's claim for refund has prescribed since the same
RULING: YES! We find for the respondent. While "taxes" and "debts" are was filed outside the thirty-day period provided for therein. According to the
distinguishable legal concepts, in certain cases as in the suit at bar, on account petitioner, the said prescriptive period commenced to run on October 14,
of their nature, the distinction becomes inconsequential. This qualification is 1947 when the denial by the Bureau of Internal Revenue of the respondent
recognized even in the United States. Thus, Palanca's claim for refund, under his letter of November 10, 1956, became
final. Considering that the case was filed with the Court of Tax Appeals only on
The term "debt" is properly used in a comprehensive sense as embracing not August 13, 1958, then it is urged that the same had prescribed.
merely money due by contract, but whatever one is bound to render to another,
either for contract or the requirements of the law. (Camden vs. Fink Coule and Taxes withheld by La Tondeña Inc. from Mr. Palanca's wages P13,172.41
Coke Co., 61 ALR 584).
Payment under Income Tax Receipt No. 677395 dated May 11,
In our jurisdiction, the rule is settled that although taxes already due have
1956 3,939.89
not, strictly speaking, the same concept as debts, they are, however
obligations that may be considered as such. (Sambrano vs. Court of Tax Payment under Income Tax Receipt No. 742334 dated August
Appeals, G.R. no. L-8652, March 30, 1957). In a more recent 14, 1956 3,939.89
case Commissioner of Internal Revenue vs. Prieto, G.R. No. L-13912,
September 30, 1960, we explicitly announced that while the distinction between P21,952.01
"taxes" and "debts" was recognized in this jurisdiction, the variance in their
legal conception does not extend to the interests paid on them, at least The petitioner also invokes prescription, at least with respect to the
insofar as Section 30 (b) (1) of the National Internal Revenue Code is sum of P17,112.21, under Section 306 of the Tax Code.2 He claims
concerned. Thus, that for the calendar year 1955, respondent Palanca paid his income
tax as follows:
Under the law, for interest to be deductible, it must be shown that there be
an indebtedness, that there should be interest upon it, and that what is PETITIONER CONTENTION: the amounts paid by claimant Palanca under
claimed as an interest deduction should have been paid or accrued within his withheld tax and under Receipt No. 677395 dated May 11, 1956 may no
the year. longer be refunded since the claim therefor was filed in court only on
August 13, 1958, or beyond two years of their payment.
APPLICATION
COURT: We find the petitioner's contention on prescription untenable.
It is here conceded that the interest paid by respondent was in consequence
of the late payment of her donor's tax, and the same was paid within the In the first place, the 30-day period under Section 11 of Republic Act 1125 did
year it is sought to be deducted. not even commence to run in this incident. It should be recalled that while the
herein petitioner originally assessed the respondent-claimant for alleged
ISSUE: whether or not such interest was paid upon an indebtedness within the gift tax liabilities, the said assessment was subsequently abandoned and in
contemplation of Section 30(b) (1) of the Tax Code. its lieu, a new one was prepared and served on the respondent-taxpayer.
The pertinent part of which reads: In this new assessment, the petitioner charged the said respondent with an
Sec. 30. Deductions from gross income — In computing net income there shall entirely new liability and for a substantially different amount from the first.
be allowed as deductions — While initially the petitioner assessed the respondent for donee's gift tax in the
amount of P170,002.74, in the subsequent assessment the latter was asked to
xxx xxx xxx pay P191,591.62 for delinquent estate and inheritance tax. Considering that it is
the interest paid on this latter-assessed estate and inheritance tax that
"Interest:
respondent Palanca is claiming refund for, then the thirty-day period under the
(1) In general. — The amount of interest paid within the taxable year on abovementioned section of Republic Act 1125 should be computed from the
indebtedness, except on indebtedness incurred or continued to purchase or carry receipt of the final denial by the Bureau of Internal Revenue of the said claim.
obligations the interest upon which is exempt from taxation as income under As has earlier been recited, respondent Palanca's claim in this incident was filed
this Title. with the Court of Tax Appeals even before it had been denied by the herein
petitioner or the Bureau of Internal Revenue. The case was filed with the said
The term "indebtedness" as used in the Tax Code of the United States court on August 13, 1958 while the petitioner denied the claim subject of the
containing similar provisions as in the above-quoted section has been defined said case only on July 24, 1959.
as the unconditional and legally enforceable obligation for the payment of
money. (Federal Taxes Vol. 2, p. 13, 019, Prentice Hall, Inc.; Mertens' Law of In the second place, the claim at bar refers to the alleged overpayment by
Federal Income Taxation, Vol. 4, p. 542.) Within the meaning of that definition, respondent Palanca of his 1955 income tax. Inasmuch as the said account was
it is apparent that a tax may be considered an indebtedness. . . . (Emphasis paid by him by installment, then the computation of the two-year prescriptive
supplied) period, under Section 306 of the National Internal Revenue Code, should be
from the date of the last installment. (Antonio Prieto, et al. vs. Collector of
"It follows that the interest paid by herein respondent for the late payment of Internal Revenue, G.R. No. L-11976, August 29, 1961) Respondent Palanca
her donor's tax is deductible from her gross income under section 30 (b) of the paid the last installment on his 1955 income tax account on August 14, 1956.
Tax Code above-quoted." His claim for refund of the alleged overpayment on it was filed with the court
We do not see any element in this case which can justify a departure from on August 13, 1958. It was, therefore, still timely instituted.
or abandonment of the doctrine in the Prieto case above.
In both this and the said case, the taxpayer sought the allowance as deductible
items from the gross income of the amounts paid by them as interests on
delinquent tax liabilities. Of course, what was involved in the cited case was the
donor's tax while the present suit pertains to interest paid on the estate and
inheritance tax. This difference, however, submits no appreciable
consequence to the rationale of this Court's previous determination that
interests on taxes should be considered as interests on indebtedness within
the meaning of Section 30(b) (1) of the Tax Code. The interpretation we have
placed upon the said section was predicated on the congressional intent, not on
the nature of the tax for which the interest was paid.
RE: PRESCRIPTION (NOT PRESCRIBED)
COURT: There were actually two claims for refund filed by the herein
respondent, Carlos Palanca, Jr., anent the case at bar.
The first one was on November 10, 1956, when he filed a claim for refund on
the interest paid by him on the donee's gift tax of P17,885.10, as originally
demanded by the Bureau of Internal Revenue.
The second one was the one filed by him on August 12, 1958, which was a
claim for refund on the interest paid by him on the estate and inheritance tax
assessed by the same Bureau in the amount of P20,624.01.
CHINA BANKING CORPORATION, petitioner, a loss from the sale or exchange, on the last day of such taxable year, of
vs. capital assets."
COURT OF APPEALS, COMMISSIONER OF INTERNAL REVENUE
The above provision conveys that the loss sustained by the holder of the
and COURT OF TAX APPEALS, respondents.
securities, which are capital assets (to him), is to be treated as a capital loss as
FACTS: if incurred from a sale or exchange transaction.
- Sometime in 1980, petitioner China Banking Corporation made a 53% A capital gain or a capital loss normally requires the concurrence of two
equity investment in the First CBC Capital (Asia) Ltd., a Hongkong conditions for it to result: (1) There is a sale or exchange; and (2) the thing sold
subsidiary engaged in financing and investment with "deposit-taking" function. or exchanged is a capital asset. When securities become worthless, there is
The investment amounted to P16,227,851.80, consisting of 106,000 shares strictly no sale or exchange but the law deems the loss anyway to be "a loss
with a par Value of P100 per share. from the sale or exchange of capital assets."A similar kind of treatment is
given, by the NIRC on the retirement of certificates of indebtedness with
- In the course of the regular examination of the financial books and investment interest coupons or in registered form, short sales and options to buy or sell
portfolios of petitioner conducted by Bangko Sentral in 1986, it was shown property where no sale or exchange strictly exists.6 In these cases, the NIRC
that First CBC Capital (Asia), Ltd., has become insolvent. With the dispenses, in effect, with the standard requirement of a sale or exchange for the
approval of Bangko Sentral, petitioner wrote-off as being worthless its application of the capital gain and loss provisions of the code.
investment in First CBC Capital (Asia), Ltd., in its 1987 Income Tax
Return and treated it as a bad debt or as an ordinary loss deductible from Capital losses are allowed to be deducted only to the extent of capital gains,
its gross income. i.e., gains derived from the sale or exchange of capital assets, and not from
any other income of the taxpayer.
- Respondent CIR disallowed the deduction and assessed petitioner for
income tax deficiency in the amount of P8,533,328.04, inclusive of In the case at bar, First CBC Capital (Asia), Ltd., the investee corporation, is a
surcharge, interest and compromise penalty. subsidiary corporation of petitioner bank whose shares in said investee
corporation are not intended for purchase or sale but as an investment.
The disallowance of the deduction was made on the ground that the Unquestionably then, any loss therefrom would be a capital loss, not an
investment should not be classified as being "worthless" and that, although ordinary loss, to the investor.
the Hongkong Banking Commissioner had revoked the license of First
CBC Capital as a "deposit-taping" company, the latter could still exercise, Section 29(d)(4)(A), of the NIRC expresses:
however, its financing and investment activities. Assuming that the
"(A) Limitations. - Losses from sales or exchanges of capital assets shall be
securities had indeed become worthless, respondent Commissioner of
allowed only to the extent provided in Section 33."
Internal Revenue held the view that they should then be classified as
"capital loss," and not as a bad debt expense there being no The pertinent provisions of Section 33 of the NIRC referred to in the
indebtedness to speak of between petitioner and its subsidiary. aforesaid Section 29(d)(4)(A), read:
Petitioner contested the ruling of respondent Commissioner before the CTA. "Section 33. Capital gains and losses. -
CTA: sustained the Commissioner, holding that the securities had not indeed "x x x xxx xxx
become worthless and ordered petitioner to pay its deficiency income tax for
1987 of P8,533,328.04 plus 20% interest per annum until fully paid. "(c) Limitation on capital losses. - Losses from sales or exchange of
capital assets shall be allowed only to the extent of the gains from such
CA: upheld the decisions of CTA sales or exchanges. If a bank or trust company incorporated under the laws
of the Philippines, a substantial part of whose business is the receipt of
ISSUE:
deposits, sells any bond, debenture, note, or certificate or other evidence
WON the shares of stock that have been declared as worthless be deducted of indebtedness issued by any corporation (including one issued by a
from petitioner’s gross income. government or political subdivision thereof), with interest coupons or in
registered form, any loss resulting from such sale shall not be subject to
COURT: PETITION MUST FAIL the foregoing limitation and shall not be included in determining the
The claim of petitioner that the shares of stock in question have become applicability of such limitation to other losses."
worthless is based on a Profit and Loss Account for the Year-End 31 December The exclusionary clause found in the foregoing text of the law does not include
1987, and the recommendation of Bangko Sentral that the equity investment be all forms of securities but specifically covers only bonds, debentures, notes,
written-off due to the insolvency of the subsidiary. While the matter may not be certificates or other evidence of indebtedness, with interest coupons or in
indubitable (considering that certain classes of intangibles, like franchises and registered form, which are the instruments of credit normally dealt with in the
goodwill, are not always given corresponding values in financial statements), usual lending operations of a financial institution. Equity holdings cannot
there may really be no need, however, to go of length into this issue since, even come close to being, within the purview of "evidence of indebtedness"
to assume the worthlessness of the shares, the deductibility thereof would still under the second sentence of the aforequoted paragraph. Verily, it is for a like
be nil in this particular case. thesis that the loss of petitioner bank in its equity investment in the
Subject to certain exceptions, such as the compensation income of individuals Hongkong subsidiary cannot also be deductible as a bad debt. The shares
and passive income subject to final tax, as well as income of non-resident aliens of stock in question do not constitute a loan extended by it to its subsidiary
and foreign corporations not engaged in trade or business in the Philippines, the (First CBC Capital) or a debt subject to obligatory repayment by the latter,
tax on income is imposed on the net income allowing certain specified essential elements to constitute a bad debt, but a long term investment
deductions from gross income to be claimed by the taxpayer. Among the made by CBC.
deductible items allowed by the National Internal Revenue Code ("NIRC") One other item. Section 34(c)(1) of the NIRC , states that the entire amount of
are bad debts and losses.2 the gain or loss upon the sale or exchange of property, as the case may be, shall
An equity investment is a capital, not ordinary, asset of the investor the sale or be recognized. The complete text reads:
exchange of which results in either a capital gain or a capital loss. The gain or "SECTION 34. Determination of amount of and recognition of gain or loss.-
the loss is ordinary when the property sold or exchanged is not a capital asset.
A capital asset is defined negatively in Section 33(1) of the NIRC; viz: "(a) Computation of gain or loss. - The gain from the sale or other
disposition of property shall be the excess of the amount realized
(1) Capital assets. - The term 'capital assets' means property held by the therefrom over the basis or adjusted basis for determining gain and
taxpayer (whether or not connected with his trade or business), but does the loss shall be the excess of the basis or adjusted basis for
not include stock in trade of the taxpayer or other property of a kind which determining loss over the amount realized. The amount realized
would properly be included in the inventory of the taxpayer if on hand at from the sale or other disposition of property shall be to sum of
the close of the taxable year, or property held by the taxpayer primarily for money received plus the fair market value of the property (other
sale to customers in the ordinary course of his trade or business, or than money) received. (As amended by E.O. No. 37)
property used in the trade or business, of a character which is subject to the
allowance for depreciation provided in subsection (f) of section twenty- "(b) Basis for determining gain or loss from sale or disposition of
nine; or real property used in the trade or business of the taxpayer." property. - The basis of property shall be - (1) The cost thereof in
cases of property acquired on or before March 1, 1913, if such
Thus, shares of stock; like the other securities defined in Section 20(t)4 of the property was acquired by purchase; or
NIRC, would be ordinary assets only to a dealer in securities or a person
engaged in the purchase and sale of, or an active trader (for his own "(2) The fair market price or value as of the date of
account) in, securities. Section 20(u) of the NIRC defines a dealer in securities acquisition if the same was acquired by inheritance; or
thus:
"(3) If the property was acquired by gift the basis shall be
"(u) The term 'dealer in securities' means a merchant of stocks or securities, the same as if it would be in the hands of the donor or the
whether an individual, partnership or corporation, with an established place last preceding owner by whom it was not acquired by
of business, regularly engaged in the purchase of securities and their resale gift, except that if such basis is greater than the fair
to customers; that is, one who as a merchant buys securities and sells them market value of the property at the time of the gift, then
to customers with a view to the gains and profits that may be derived for the purpose of determining loss the basis shall be
therefrom." such fair market value; or
In the hands, however, of another who holds the shares of stock by way of an "(4) If the property, other than capital asset referred to in
investment, the shares to him would be capital assets. When the shares held Section 21 (e), was acquired for less than an adequate
by such investor become worthless, the loss is deemed to be a loss from the consideration in money or moneys worth, the basis of
sale or exchange of capital assets. Section 29(d)(4)(B) of the NIRC states: such property is (i) the amount paid by the transferee for
the property or (ii) the transferor's adjusted basis at the
"(B) Securities becoming worthless. - If securities as defined in Section 20 time of the transfer whichever is greater.
become worthless during the tax" year and are capital assets, the loss
resulting therefrom shall, for the purposes of his Title, be considered as
"(5) The basis as defined in paragraph (c) (5) of this PHILIPPINE REFINING COMPANY (now known as "UNILEVER
section if the property was acquired in a transaction PHILIPPINES [PRC], INC."), petitioner,
where gain or loss is not recognized under paragraph (c) vs.
(2) of this section. (As amended by E.O. No. 37) COURT OF APPEALS, COURT OF TAX APPEALS, and THE
COMMISSIONER OF INTERNAL REVENUE, respondents.
"(c) Exchange of property.
FACTS: Petitioner Philippine Refining Company (PRC) was assessed by
"(1) General rule.- Except as herein provided, upon the respondent Commissioner of Internal Revenue (Commissioner) to pay a
sale or exchange of property, the entire amount of the deficiency tax for the year 1985 in the amount of P1,892,584.00, computed as
gain or loss, as the case may be, shall be recognized. follows:
"(2) Exception. - No gain or loss shall be recognized if in Deficiency Income Tax
pursuance of a plan of merger or consolidation (a) a
corporation which is a party to a merger or consolidation Net Income per investigation P197,502,568.00
exchanges property solely for stock in a corporation Add: Disallowances
which is, a party to the merger or consolidation, (b) a Bad Debts P 713,070.93
shareholder exchanges stock in a corporation which is a Interest Expense P 2,666,545.49
party to the merger or consolidation solely for the stock —————— ——————
in another corporation also a party to the merger or P3,379,616.00
consolidation, or (c) a security holder of a corporation
Net Taxable Income 200,882,184.00
which is a party to the merger or consolidation exchanges
his securities in such corporation solely for stock or Tax Due Thereon 70,298,764.00
securities in another corporation, a party to the merger or Less: Tax Paid 69,115,899.00
consolidation. Deficiency Income Tax 1,182,865.00
Add: 20% Interest (60% max.) 709,719.00
"No gain or loss shall also be recognized if property is transferred to a
——————
corporation by a person in exchange for stock in such corporation of which as a
result of such exchange said person, alone or together with others, not Total Amount Due and Collectible P1,892,584.002
exceeding four persons, gains control of said corporation: Provided, That stocks
issued for services shall not be considered as issued in return of property." The assessment was timely protested by petitioner on April 26, 1989, on the
ground that it was based on the erroneous disallowances of "bad debts"
The above law should be taken within context on the general subject of the and "interest expense" although the same are both allowable and legal
determination, and recognition of gain or loss; it is not preclusive of, let alone deductions.
renders completely inconsequential, the more specific provisions of the code.
Thus, pursuant, to the same section of the law, no such recognition shall be Respondent Commissioner, however, issued a warrant of garnishment
made if the sale or exchange is made in pursuance of a plan of corporate merger against the deposits of petitioner at a branch of City Trust Bank, in Makati,
or consolidation or, if as a result of an exchange of property for stocks, the Metro Manila, which action the latter considered as a denial of its protest.
exchanger, alone or together with others not exceeding four, gains control of the Petitioner accordingly filed a petition for review with the Court of Tax
corporation.7 Then, too, how the resulting gain might be taxed, or whether or Appeals (CTA) on the same assignment of error, that is, that the "bad debts"
not the loss would be deductible and how, are matters properly dealt with and "interest expense" are legal and allowable deductions.
elsewhere in various other sections of the NIRC.8 At all events, it may not be
amiss to once again stress that the basic rule is still that any capital loss can be CTA: modified the findings of the Commissioner by reducing the deficiency
deducted only from capital gains under Section 33(c) of the NIRC. income tax assessment to P237,381.26, with surcharge and interest incident to
delinquency.
In sum -
- reversed and set aside the Commissioner's disallowance of the
(a) The equity investment in shares of stock held by CBC of interest expense of P2,666,545.19 but maintained the
approximately 53% in its Hongkong subsidiary, the First CBC disallowance of the supposed bad debts of thirteen (13) debtors
Capital (Asia), Ltd., is not an indebtedness, and it is a capital, not an in the total sum of P395,324.27.
ordinary, asset.9 1âwphi1
CA: denied due course to the petition for review and dismissed the same on
(b) Assuming that the equity investment of CBC has indeed become August 24, 1994 in CA-G.R. SP No. 31190,4 on the following ratiocination:
"worthless," the loss sustained is a capital, not an ordinary, loss.10
We agree with respondent Court of Tax Appeals:
(c) The capital loss sustained by CBC can only be deducted from
capital gains if any derived by it during the same taxable year that Out of the sixteen (16) accounts alleged as bad debts, We find that only three
the securities have become "worthless."11 (3) accounts have met the requirements of the worthlessness of the
accounts, hence were properly written off as: bad debts: TOTAL P
317,746.66
xxx xxx xxx
With regard to the other accounts: TOTAL P 395,324.27
We find that said accounts have not satisfied the requirements of the
"worthlessness of a debt".
Mere testimony of the Financial Accountant of the Petitioner explaining the
worthlessness of said debts is seen by this Court as nothing more than a self-
serving exercise which lacks probative value. There was no iota of
documentary evidence (e.g., collection letters sent, report from investigating
fieldmen, letter of referral to their legal department, police report/affidavit that
the owners were bankrupt due to fire that engulfed their stores or that the owner
has been murdered. etc.), to give support to the testimony of an employee of the
Petitioner. Mere allegations cannot prove the worthlessness of such debts in
1985. Hence, the claim for deduction of these thirteen (13) debts should be
rejected.5
CA RELIANCE: Collector vs. Goodrich International Rubber Co.,6 which
established the rule in determining the "worthlessness of a debt." In said case,
we held that for debts to be considered as "worthless," and thereby qualify
as "bad debts" making them deductible, the taxpayer should show that
(1) there is a valid and subsisting debt.
(2) the debt must be actually ascertained to be worthless and uncollectible
during the taxable year;
(3) the debt must be charged off during the taxable year; and
(4) the debt must arise from the business or trade of the taxpayer.

Additionally, before a debt can be considered worthless, the taxpayer must also
show that it is indeed uncollectible even in the future.
Furthermore, there are steps outlined to be undertaken by the taxpayer to
prove that he exerted diligent efforts to collect the debts, viz.: (1) sending of
statement of accounts; (2) sending of collection letters; (3) giving the account to
a lawyer for collection; and (4) filing a collection case in court.
On the foregoing considerations, respondent Court of Appeals held that
petitioner did not satisfy the requirements of "worthlessness of a debt" as to the
thirteen (13) accounts disallowed as deductions.
It appears that the only evidentiary support given by PRC for its aforesaid
claimed deductions was the explanation or justification posited by its financial
adviser or accountant, Guia D. Masagana. Her allegations were not supported
by any documentary evidence, hence both the Court of Appeals and the CTA
ruled that said contentions per se cannot prove that the debts were indeed PRC CONTENTION: that nobody is in a better position to determine when
uncollectible and can be considered as bad debts as to make them an obligation becomes a bad debt than the creditor itself, and that its
deductible. That both lower courts are correct is shown by petitioner's own judgment should not be substituted by that of respondent court as it is PRC
submission and the discussion thereof which we have taken time and patience which has the facilities in ascertaining the collectibility or uncollectibility
to cull from the antecedent proceedings in this case, albeit bordering on factual of these debts, are presumptuous and uncalled for.
settings.
COURT: The Court of Tax Appeals is a highly specialized body specifically
ISSUES: created for the purpose of reviewing tax cases. Through its expertise, it is
undeniably competent to determine the
1. WON bad debts requirements are met to be deductible as assessed by issue of whether or not the debt is deductible through the evidence presented
the CA = NO! before it.8

2. WON PRC should be liable for penalties and interests = YES! Because of this recognized expertise, the findings of the CTA will not
ordinarily be reviewed absent a showing of gross error or abuse on its
part.9 The findings of fact of the CTA are binding on this Court and in the
RULING (1) absence of strong reasons for this Court to delve into facts, only questions of
law are open for determination. 10 Were it not, therefore, due to the desire of
The accounts of Remoblas Store in the amount of P11,961.00 and CM Variety this Court to satisfy petitioner's calls for clarification and to use this case as a
Store in the amount of P10,895.82 are uncollectible, according to petitioner, vehicle for exemplification, this appeal could very well have been summarily
since the stores were burned in November, 1984 and in early 1985, dismissed.
respectively, and there are no assets belonging to the debtors that can be
garnished by PRC.7 PETITIONER THESIS: that despite the supervening delay in the tax
payment, nothing is lost on the part of the Government because in the event
COURT: However, PRC failed to show any documentary evidence for said that these debts are collected, the same will be returned as taxes to it in the
allegations. Not a single document was offered to show that the stores were year of the recovery.
burned, even just a police report or an affidavit attesting to such loss by fire. In
fact, petitioner did not send even a single demand letter to the owners of said COURT: This is an irresponsible statement which deliberately ignores the fact
stores. that while the Government may eventually recover revenues under that
hypothesis, the delay caused by the non-payment of taxes under such a
The account of Tomas Store in the amount of P16,842.79 is uncollectible, contingency will obviously have a disastrous effect on the revenue
claims petitioner PRC, since the owner thereof was murdered and left no collections necessary for governmental operations during the period
visible assets which could satisfy the debt. concerned.
COURT: Withal, just like the accounts of the two other stores just mentioned, 2. PETITIONER ARGUES: that the imposition of the 25% surcharge and
petitioner again failed to present proof of the efforts exerted to collect the the 20% delinquency interest due to delay in its payment of the tax
debt, other than the aforestated asseverations of its financial adviser. assessed is improper and unwarranted, considering that the assessment of the
Commissioner was modified by the CTA and the decision of said court has not
The accounts of Aboitiz Shipping Corporation and J. Ruiz Trucking in the
yet become final and executory.
amounts of P89,483.40 and P69,640.34, respectively, both of which allegedly
arose from the hijacking of their cargo and for which they were given 30% COURT: Regarding the 25% surcharge penalty, Section 248 of the Tax Code
rebates by PRC, are claimed to be uncollectible. provides:
COURT: Again, petitioner failed to present an iota of proof, not even a copy Sec. 248. Civil Penalties. — (a) There shall be imposed, in addition to the tax
of the supposed policy regulation of PRC that it gives rebates to clients in case required to be paid, a penalty equivalent to twenty-five percent (25%) of the
of loss arising from fortuitous events or force majeure, which rebates it now amount due, in the following cases:
passes off as uncollectible debts.
xxx xxx xxx
As to the account of P13,550.00 representing the balance collectible from
Renato Alejandro, a former employee who failed to pay the judgment against (3) Failure to pay the tax within the time prescribed for its payment.
him, it is petitioner's theory that the same can no longer be collected since his With respect to the penalty of 20% interest, the relevant provision is found in
whereabouts are unknown and he has no known property which can be Section 249 of the same Code, as follows:
garnished or levied upon.
Sec. 249. Interest. — (a) In general. — There shall be assessed and collected on
COURT: Once again, petitioner failed to prove the existence of the said case any unpaid amount of tax, interest at the rate of twenty percent (20%) per
against that debtor or to submit any documentation to show that Alejandro was annum, or such higher rate as may be prescribed by regulations, from the date
indeed bound to pay any judgment obligation. prescribed for payment until the amount is fully paid.
The amount of P13,772.00 corresponding to the debt of Lucito Sta. Maria is xxx xxx xxx
allegedly due to the loss of his stocks through robbery and the account is
uncollectible due to his insolvency. (c) Delinquency interest. — In case of failure pay:

COURT: Petitioner likewise failed to submit documentary evidence, not even (1) The amount of the tax due on any return required to be
the written reports of the alleged investigation conducted by its agents as filed, or
testified to by its aforenamed financial adviser. (2) The amount of the tax due for which no return is required, or
Regarding the accounts of C. Itoh in the amount of P19,272.22, Crocklaan B.V. (3) A deficiency tax, or any surcharge or interest thereon, on the due date
in the sum of P77,690.00, and Craig, Mostyn Pty. Ltd. with a balance of appearing in the notice and demand of the Commissioner,
P23,738.00, petitioner contends that these debtors being foreign
corporations, it can sue them only in their country of incorporation; and there shall be assessed and collected, on the unpaid amount, interest at the rate
since this will entail expenses more than the amounts of the debts to be prescribed in paragraph (a) hereof until the amount is fully paid, which interest
collected, petitioner did not file any collection suit but opted to write them shall form part of the tax. (emphasis supplied)
off as bad debts. xxx xxx xxx
COURT: Petitioner was unable to show proof of its efforts to collect the As correctly pointed out by the Solicitor General, the deficiency tax
debts, even by a single demand letter therefor. While it is not required to file assessment in this case, which was the subject of the demand letter of
suit, it is at least expected by the law to produce reasonable proof that the debts respondent Commissioner dated April 11,1989, should have been paid within
are uncollectible although diligent efforts were exerted to collect the same. thirty (30) days from receipt thereof. By reason of petitioner's default
The account of Enriched Food Corporation in the amount of P24,158.00 thereon, the delinquency penalties of 25% surcharge and interest of 20%
remains unpaid, although petitioner claims that it sent several letters. accrued from April 11, 1989.
COURT: This is not sufficient to sustain its position. even if true, but even The fact that petitioner appealed the assessment to the CTA and that the same
smacks of insouciance on its part. On top of that, it was unable to show a single was modified does not relieve petitioner of the penalties incident to
copy of the alleged demand letters sent to the said corporation or any of its delinquency. The reduced amount of P237,381.25 is but a part of the original
corporate officers. assessment of P1,892,584.00.
With regard to the account of AFPCES for unpaid supplies in the amount of We have likewise explained that it is mandatory to collect penalty and interest
P13,833.62, petitioner asserts that since the debtor is an agency of the at the stated rate in case of delinquency. The intention of the law is to
government, PRC did not file a collection suit therefor. discourage delay in the payment of taxes due the Government and, in this
sense, the penalty and interest are not penal but compensatory for the
COURT: Yet, the mere fact that AFPCES is a government agency does not concomitant use of the funds by the taxpayer beyond the date when he is
preclude PRC from filing suit since said agency, while discharging supposed to have paid them to the Government. 12 Unquestionably, petitioner
proprietary functions, does not enjoy immunity from suit. Such pretension of chose to turn a deaf ear to these injunctions.
petitioner cannot pass judicial muster.
PETITION DENIED.
No explanation is offered by petitioner as to why the unpaid account of U'
Ren Mart Enterprise in the amount of P10,487.08 was written off as a bad
debt.
However, the decision of the CTA includes this debtor in its findings on the
lack of documentary evidence to justify the deductions claimed, since the
worthlessness of the debts involved are sought to be established by the mere
self-serving testimony of its financial consultant.
BASILAN ESTATES, INC., petitioner, (1)In general. — A reasonable allowance for deterioration of
vs. property arising out of its use or employment in the business or
THE COMMISSIONER OF INTERNAL REVENUE and THE COURT trade, or out of its not being used: Provided, That when the
OF TAX APPEALS, respondents. allowance authorized under this subsection shall equal the capital
invested by the taxpayer . . . no further allowance shall be made. . . .
FACTS:
- A Philippine corporation engaged in the coconut industry, Basilan Estates, allows a deduction from gross income for depreciation but limits the recovery
to the capital invested in the asset being depreciated.
Inc., with principal offices in Basilan City, filed on March 24, 1954 its
income tax returns for 1953 and paid an income tax of P8,028. The income tax law does not authorize the depreciation of an asset beyond
- On February 26, 1959, the Commissioner of Internal Revenue, per its acquisition cost. Hence, a deduction over and above such cost cannot be
claimed and allowed. The reason is that deductions from gross income are
examiners' report of February 19, 1959, assessed Basilan Estates, Inc., a privileges,5 not matters of right.6 They are not created by implication but
deficiency income tax of P3,912 for 1953 and P86,876.85 as 25% surtax on upon clear expression in the law.
unreasonably accumulated profits as of 1953 pursuant to Section 25 of the
Tax Code. Moreover, the recovery, free of income tax, of an amount more than the
- On non-payment of the assessed amount, a warrant of distraint and levy was invested capital in an asset will transgress the underlying purpose of a
depreciation allowance. For then what the taxpayer would recover will be,
issued but the same was not executed because Basilan Estates, Inc. not only the acquisition cost, but also some profit.
succeeded in getting the Deputy Commissioner of Internal Revenue to order
the Director of the district in Zamboanga City to hold execution and Recovery in due time thru depreciation of investment made is the
maintain constructive embargo instead. Because of its refusal to waive the philosophy behind depreciation allowance; the idea of profit on the
period of prescription, the corporation's request for reinvestigation was not investment made has never been the underlying reason for the allowance of
given due course, and on December 2, 1960, notice was served the a deduction for depreciation.
corporation that the warrant of distraint and levy would be executed.
Accordingly, the claim for depreciation beyond P36,842.04 or in the amount of
- On December 20, 1960, Basilan Estates, Inc. filed before the Court of P10,500.49 has no justification in the law. The determination, therefore, of the
Tax Appeals a petition for review of the Commissioner's assessment, Commissioner of Internal Revenue disallowing said amount, affirmed by the
alleging prescription of the period for assessment and collection; error in Court of Tax Appeals, is sustained.
disallowing claimed depreciations, travelling and miscellaneous
B. Expenses. — The next item involves disallowed expenses incurred in 1953,
expenses; and error in finding the existence of unreasonably accumulated
broken as follows:
profits and the imposition of 25% surtax thereon.
CTA: there was no prescription and affirmed the deficiency assessment in toto. Miscellaneous expenses P6,759.17
ISSUES: Officer's travelling expenses 2,300.40
1. Has the Commissioner's right to collect deficiency income tax prescribed?
2. Was the disallowance of items claimed as deductible proper? Total P9,059.57
3. Have there been unreasonably accumulated profits? If so, should the 25%
surtax be imposed on the balance of the entire surplus from 1947-1953, or only These were disallowed on the ground that the nature of these expenses could
for 1953? not be satisfactorily explained nor could the same be supported by appropriate
papers.
4. Is the petitioner exempt from the penalty tax under Republic Act 1823
amending Section 25 of the Tax Code? PETITIONER’S ARGUMENT:

COURT:
- Felix Gulfin, petitioner's accountant, explained the P6,759.17 was actual
expenses credited to the account of the president of the corporation incurred
RE: DEDUCTIONS in the interest of the corporation during the president's trip to Manila; he
stated that the P2,300.40 was the president's travelling expenses to and from
A. Depreciation. — Basilan Estates, Inc. claimed deductions for the
Manila as to the vouchers and receipts of these, he said the same were made
depreciation of its assets up to 1949 on the basis of their acquisition cost. As of
but got burned during the Basilan fire on March 30, 1962.
January 1, 1950 it changed the depreciable value of said assets by
increasing it to conform with the increase in cost for their replacement. - Petitioner further argues that when it sent its records to Manila in February,
Accordingly, from 1950 to 1953 it deducted from gross income the value of 1959, the papers in support of these miscellaneous and travelling expenses
depreciation computed on the reappraised value. were not included for the reason that by February 9, 1959, when the Bureau
of Internal Revenue decided to investigate, petitioner had no more
In 1953, the year involved in this case, taxpayer claimed the following
obligation to keep the same since five years had lapsed from the time these
depreciation deduction:
expenses were incurred.
Reappraised assets P47,342.53 On this ground, the petitioner may be sustained, for under Section 337 of
the Tax Code, receipts and papers supporting such expenses need be kept
New assets consisting of hospital building and by the taxpayer for a period of five years from the last entry. At the time of
equipment 3,910.45 the investigation, said five years had lapsed. Taxpayer's stand on this issue
is therefore sustained.
Total depreciation
P51,252.98

COMMISSIONER:
- Upon investigation and examination of taxpayer's books and papers, the
Commissioner of Internal Revenue found that the reappraised assets
depreciated in 1953 were the same ones upon which depreciation was
claimed in 1952. And for the year 1952, the Commissioner had already
determined, with taxpayer's concurrence, the depreciation allowable on said
assets to be P36,842.04, computed on their acquisition cost at rates fixed by
the taxpayer. Hence, the Commissioner pegged the deductible
depreciation for 1953 on the same old assets at P36,842.04 and
disallowed the excess thereof in the amount of P10,500.49.
ISSUE:
WON depreciation shall be determined on the acquisition cost or on the
reappraised value of the assets.
COURT: ACQUISITION COST
Depreciation is the gradual diminution in the useful value of tangible property
resulting from wear and tear and normal obsolescense. The term is also applied
to amortization of the value of intangible assets, the use of which in the trade or
business is definitely limited in duration.2 Depreciation commences with the
acquisition of the property and its owner is not bound to see his property
gradually waste, without making provision out of earnings for its
replacement. It is entitled to see that from earnings the value of the
property invested is kept unimpaired, so that at the end of any given term
of years, the original investment remains as it was in the beginning. It is not
only the right of a company to make such a provision, but it is its duty to its
bond and stockholders, and, in the case of a public service corporation, at least,
its plain duty to the public.3 Accordingly, the law permits the taxpayer to
recover gradually his capital investment in wasting assets free from income
tax.4 Precisely, Section 30 (f) (1) which states:
FERNANDEZ HERMANOS, INC. vs. CIR (a) Allowance of losses in Mati Lumber Co. (1950). — The Commissioner of
Internal Revenue questions the Tax Court's allowance of the taxpayer's writing
FACTS: off as worthless securities in its 1950 return the sum of P8,050.00 representing
the cost of shares of stock of Mati Lumber Co. acquired by the taxpayer on
Cases L-21551 and L-21557 January 1, 1948, on the ground that the worthlessness of said stock in the year
The taxpayer, Fernandez Hermanos, Inc., is a domestic corporation 1950 had not been clearly established. The Commissioner contends that
organized for the principal purpose of engaging in business as an "investment although the said Company was no longer in operation in 1950, it still had its
company" with main office at Manila. sawmill and equipment which must be of considerable value. The Court,
however, found that "the company ceased operations in 1949 when its Manager
Upon verification of the taxpayer's income tax returns for the period in and owner, a certain Mr. Rocamora, left for Spain ,where he subsequently died.
question, the Commissioner of Internal Revenue assessed against the When the company eased to operate, it had no assets, in other words,
taxpayer the sums of P13,414.00, P119,613.00, P11,698.00, P6,887.00 and completely insolvent. This information as to the insolvency of the Company —
P14,451.00 as alleged deficiency income taxes for the years 1950, 1951, 1952, reached (the taxpayer) in 1950," when it properly claimed the loss as a
1953 and 1954, respectively. deduction in its 1950 tax return, pursuant to Section 30(d) (4) (b) or Section 30
Said assessments were the result of alleged discrepancies found upon the (e) (3) of the National Internal Revenue Code. 2
examination and verification of the taxpayer's income tax returns for the COURT: We find no reason to disturb this finding of the Tax Court. There
said years, summarized by the Tax Court in its decision of June 10, 1963 in was adequate basis for the writing off of the stock as worthless securities.
CTA Case No. 787, as follows:
Assuming that the Company would later somehow realize some proceeds from
1. Losses — its sawmill and equipment, which were still existing as claimed by the
a. Losses in Mati Lumber Co. (1950) P 8,050.00 Commissioner, and that such proceeds would later be distributed to its
b. Losses in or bad debts of Palawan Manganese Mines, Inc. (1951) stockholders such as the taxpayer, the amount so received by the taxpayer
353,134.25 would then properly be reportable as income of the taxpayer in the year it
c. Losses in Balamban Coal Mines — is received.
1950 8,989.76
(b) Disallowance of losses in or bad debts of Palawan Manganese Mines,
1951 27,732.66 Inc. (1951). — The taxpayer appeals from the Tax Court's disallowance of its
writing off in 1951 as a loss or bad debt the sum of P353,134.25, which it had
d. Losses in Hacienda Dalupiri — advanced or loaned to Palawan Manganese Mines, Inc. The Tax Court's
1950 17,418.95 findings on this item follow:
Sometime in 1945, Palawan Manganese Mines, Inc., the controlling
1951 29,125.82
stockholders of which are also the controlling stockholders of
1952 26,744.81 petitioner corporation, requested financial help from petitioner to
enable it to resume it mining operations in Coron, Palawan. The
1953 21,932.62 request for financial assistance was readily and unanimously
approved by the Board of Directors of petitioner, and thereafter a
1954 42,938.56 memorandum agreement was executed on August 12, 1945,
e. Losses in Hacienda Samal — embodying the terms and conditions under which the financial
assistance was to be extended, the pertinent provisions of which are
1951 8,380.25 as follows:
1952 7,621.73 Pursuant to the agreement mentioned above, petitioner gave to Palawan
2. Excessive depreciation of Houses — Manganese Mines, Inc. yearly advances starting from 1945, which advances
amounted to P587,308.07 by the end of 1951. Despite these advances and the
1950 P 8,180.40 resumption of operations by Palawan Manganese Mines, Inc., it continued to
suffer losses. By 1951, petitioner became convinced that those advances could
1951 8,768.11
no longer be recovered. While it continued to give advances, it decided to write
1952 18,002.16 off as worthless the sum of P353,134.25. This amount "was arrived at on the
basis of the total of advances made from 1945 to 1949 in the sum of
1953 13,655.25 P438,981.39, from which amount the sum of P85,647.14 had to be deducted,
the latter sum representing its pre-war assets. (t.s.n., pp. 136-139, Id)." (Page 4,
1954 29,314.98 Memorandum for Petitioner.) Petitioner decided to maintain the advances given
3. Taxable increase in net worth — in 1950 and 1951 in the hope that it might be able to recover the same, as in
fact it continued to give advances up to 1952. From these facts, and as
1950 P 30,050.00 admitted by petitioner itself, Palawan Manganese Mines, Inc., was still in
operation when the advances corresponding to the years 1945 to 1949 were
1951 1,382.85
written off the books of petitioner. Under the circumstances, was the sum of
4. Gain realized from sale of real property in 1950 P 11,147.2611 P353,134.25 properly claimed by petitioner as deduction in its income tax
return for 1951, either as losses or bad debts?
Tax Court: sustained the Commissioner's disallowances of Item 1, sub-items
(b) and (e) and Item 2 of the above summary, but overruled the Commissioner's COURT: It will be noted that in giving advances to Palawan Manganese
disallowances of all the remaining items. Mine Inc., petitioner did not expect to be repaid. It is true that some
testimonial evidence was presented to show that there was some agreement that
- It therefore modified the deficiency assessments accordingly, the advances would be repaid, but no documentary evidence was presented to
found the total deficiency income taxes due from the taxpayer for this effect.
the years under review to amount to P123,436.00 instead of
P166,063.00 as originally assessed by the Commissioner, and The memorandum agreement signed by the parties appears to be very clear that
rendered the following judgment: the consideration for the advances made by petitioner was 15% of the net
profits of Palawan Manganese Mines, Inc. In other words, if there were no
RESUME earnings or profits, there was no obligation to repay those advances.
1950 P2,748.00 It has been held that the voluntary advances made without expectation of
repayment do not result in deductible losses. 1955 PH Fed. Taxes, Par. 13, 329,
1951 108,724.00 citing W. F. Young, Inc. v. Comm., 120 F 2d. 159, 27 AFTR 395; George B.
Markle, 17 TC. 1593.
1952 3,600.00
Is the said amount deductible as a bad debt? As already stated, petitioner
1953 2,501.00 gave advances to Palawan Manganese Mines, Inc., without expectation of
repayment. Petitioner could not sue for recovery under the memorandum
1954 5,863.00 agreement because the obligation of Palawan Manganese Mines, Inc. was to
pay petitioner 15% of its net profits, not the advances. No bad debt could arise
where there is no valid and subsisting debt.
Total P123,436.00
Again, assuming that in this case there was a valid and subsisting debt and
ISSUES: that the debtor was incapable of paying the debt in 1951, when petitioner
wrote off the advances and deducted the amount in its return for said year,
1. the correctness of the Tax Court's rulings with respect to the yet the debt is not deductible in 1951 as a worthless debt.
disputed items of disallowances enumerated in the Tax Court's
summary reproduced above = CORRECT! It appears that the debtor was still in operation in 1951 and 1952, as
petitioner continued to give advances in those years. It has been held that if
2. whether or not the government's right to collect the deficiency the debtor corporation, although losing money or insolvent, was still operating
income taxes in question has already prescribed. NO! at the end of the taxable year, the debt is not considered worthless and
therefore not deductible. 3
RULING:
The Tax Court's disallowance of the write-off was proper. The Solicitor
On the first issue, we will discuss the disputed items of disallowances seriatim.
General has rightly pointed out that the taxpayer has taken an "ambiguous
1. Re allowances/disallowances of losses. position " and "has not definitely taken a stand on whether the amount involved
is claimed as losses or as bad debts but insists that it is either a loss or a bad
debt." 4 We sustain the government's position that the advances made by the
taxpayer to its 100% subsidiary, Palawan Manganese Mines, Inc. amounting to "The Taxpayer deducted from its income tax returns for
P587,308,07 as of 1951 were investments and not loans. 5 The evidence on the years from 1950 to 1954 inclusive, the corresponding
record shows that the board of directors of the two companies since August, yearly losses sustained in the operation of Hacienda
1945, were identical and that the only capital of Palawan Manganese Mines, Dalupiri, which losses represent the excess of its yearly
Inc. is the amount of P100,000.00 entered in the taxpayer's balance sheet as its expenditures over the receipts; that is, the losses
investment in its subsidiary company. 6 This fact explains the liberality with represent the difference between the sales of livestock
which the taxpayer made such large advances to the subsidiary, despite the and the actual cash disbursements or expenses." (Pages
latter's admittedly poor financial condition. 21-22, Memorandum for Petitioner.)
PETITIONER CONTENTION: that its advances were loans to its As the Hacienda Dalupiri was operated by petitioner for business
subsidiary as against the Tax Court's finding that under their memorandum and since it sustained losses in its operation, which losses were
agreement, the taxpayer did not expect to be repaid, since if the subsidiary had determined by means of inventories authorized under Section 100 of
no earnings, there was no obligation to repay those advances, becomes Revenue Regulations No. 2, it was error for respondent to have
immaterial, in the light of our resolution of the question. disallowed the deduction of said losses. The same is true with
respect to loss sustained in the operation of the Hacienda Samal for
COURT: The Tax Court correctly held that the subsidiary company was still in the years 1951 and 1952. 10
operation in 1951 and 1952 and the taxpayer continued to give it advances in
those years, and, therefore, the alleged debt or investment could not properly The Commissioner questions that the losses sustained by the taxpayer were
be considered worthless and deductible in 1951, as claimed by the properly based on the inventory method of accounting. He concedes, however,
taxpayer. "that the regulations referred to does not specify how the inventories are to be
made. The Tax Court, however, felt satisfied with the evidence presented by the
Furthermore, neither under Section 30 (d) (2) of our Tax Code providing for taxpayer ... which merely consisted of an alleged physical count of the number
deduction by corporations of losses actually sustained and charged off of the livestock in Hacienda Dalupiri for the years involved." 11 The Tax Court
during the taxable year nor under Section 30 (e) (1) thereof providing for was satisfied with the method adopted by the taxpayer as a farmer
deduction of bad debts actually ascertained to be worthless and charged breeding livestock, reporting on the basis of receipts and disbursements.
off within the taxable year, can there be a partial writing off of a loss or We find no Compelling reason to disturb its findings.
bad debt, as was sought to be done here by the taxpayer. For such losses or
bad debts must be ascertained to be so and written off during the taxable year, 2. Disallowance of excessive depreciation of buildings (1950-1954). — During
are therefore deductible in full or not at all, in the absence of any express the years 1950 to 1954, the taxpayer claimed a depreciation allowance for its
provision in the Tax Code authorizing partial deductions. buildings at the annual rate of 10%. The Commissioner claimed that the
reasonable depreciation rate is only 3% per annum, and, hence, disallowed as
The Tax Court held that the taxpayer's loss of its investment in its subsidiary excessive the amount claimed as depreciation allowance in excess of 3%
could not be deducted for the year 1951, as the subsidiary was still in operation annually. We sustain the Tax Court's finding that the taxpayer did not submit
in 1951 and 1952. The taxpayer, on the other hand, claims that its advances adequate proof of the correctness of the taxpayer's claim that the depreciable
were irretrievably lost because of the staggering losses suffered by its assets or buildings in question had a useful life only of 10 years so as to justify
subsidiary in 1951 and that its advances after 1949 were "only limited to the its 10% depreciation per annum claim, such finding being supported by the
purpose of salvaging whatever ore was already available, and for the purpose record. The taxpayer's contention that it has many zero or one-peso
of paying the wages of the laborers who needed help." 7 The correctness of assets, 12 representing very old and fully depreciated assets serves but to
the Tax Court's ruling in sustaining the disallowance of the write-off in support the Commissioner's position that a 10% annual depreciation rate
1951 of the taxpayer's claimed losses is borne out by subsequent events was excessive.
shown in Cases L-24972 and L-24978 involving the taxpayer's 1957 income
tax liability. (Infra, paragraph 6.) It will there be seen that by 1956, the 3. Taxable increase in net worth (1950-1951). — The Tax Court set aside the
obligation of the taxpayer's subsidiary to it had been reduced from P587,398.97 Commissioner's treatment as taxable income of certain increases in the
in 1951 to P442,885.23 in 1956, and that it was only on January 1, 1956 that the taxpayer's net worth. It found that:
subsidiary decided to cease operations. 8
For the year 1950, respondent determined that petitioner had an increase in net
(c) Disallowance of losses in Balamban Coal Mines (1950 and 1951). — The worth in the sum of P30,050.00, and for the year 1951, the sum of P1,382.85.
Court sustains the Tax Court's disallowance of the sums of P8,989.76 and These amounts were treated by respondent as taxable income of petitioner for
P27,732.66 spent by the taxpayer for the operation of its Balamban coal mines said years.
in Cebu in 1950 and 1951, respectively, and claimed as losses in the taxpayer's
It appears that petitioner had an account with the Manila Insurance Company,
returns for said years. The Tax Court correctly held that the losses "are
the records bearing on which were lost. When its records were reconstituted the
deductible in 1952, when the mines were abandoned, and not in 1950 and 1951,
amount of P349,800.00 was set up as its liability to the Manila Insurance
when they were still in operation." 9 The taxpayer's claim that these expeditions
Company. It was discovered later that the correct liability was only 319,750.00,
should be allowed as losses for the corresponding years that they were incurred,
or a difference of P30,050.00, so that the records were adjusted so as to show
because it made no sales of coal during said years, since the promised road or
the correct liability. The correction or adjustment was made in 1950.
outlet through which the coal could be transported from the mines to the
Respondent contends that the reduction of petitioner's liability to Manila
provincial road was not constructed, cannot be sustained. Some definite event
Insurance Company resulted in the increase of petitioner's net worth to the
must fix the time when the loss is sustained, and here it was the event of actual
extent of P30,050.00 which is taxable. This is erroneous. The principle
abandonment of the mines in 1952. The Tax Court held that the losses, totalling
underlying the taxability of an increase in the net worth of a taxpayer rests on
P36,722.42 were properly deductible in 1952, but the appealed judgment does
the theory that such an increase in net worth, if unreported and not explained by
not show that the taxpayer was credited therefor in the determination of its tax
the taxpayer, comes from income derived from a taxable source. (See Perez v.
liability for said year. This additional deduction of P36,722.42 from the
Araneta, G.R. No. L-9193, May 29, 1957; Coll. vs. Reyes, G.R. Nos. L- 11534
taxpayer's taxable income in 1952 would result in the elimination of the
& L-11558, Nov. 25, 1958.) In this case, the increase in the net worth of
deficiency tax liability for said year in the sum of P3,600.00 as determined by
petitioner for 1950 to the extent of P30,050.00 was not the result of the receipt
the Tax Court in the appealed judgment.
by it of taxable income. It was merely the outcome of the correction of an error
(d) and (e) Allowance of losses in Hacienda Dalupiri (1950 to 1954) and in the entry in its books relating to its indebtedness to the Manila Insurance
Hacienda Samal (1951-1952). — The Tax Court overruled the Commissioner's Company. The Income Tax Law imposes a tax on income; it does not tax any or
disallowance of these items of losses thus: every increase in net worth whether or not derived from income. Surely, the
said sum of P30,050.00 was not income to petitioner, and it was error for
Petitioner deducted losses in the operation of its Hacienda Dalupiri the sums of respondent to assess a deficiency income tax on said amount.
P17,418.95 in 1950, P29,125.82 in 1951, P26,744.81 in 1952, P21,932.62 in
1953, and P42,938.56 in 1954. These deductions were disallowed by The same holds true in the case of the alleged increase in net worth of petitioner
respondent on the ground that the farm was operated solely for pleasure or as a for the year 1951 in the sum of P1,382.85. It appears that certain items (all
hobby and not for profit. This conclusion is based on the fact that the farm was amounting to P1,382.85) remained in petitioner's books as outstanding
operated continuously at a loss.1awphîl.nèt liabilities of trade creditors. These accounts were discovered in 1951 as having
been paid in prior years, so that the necessary adjustments were made to correct
From the evidence, we are convinced that the Hacienda Dalupiri was operated the errors. If there was an increase in net worth of the petitioner, the increase in
by petitioner for business and not pleasure. It was mainly a cattle farm, net worth was not the result of receipt by petitioner of taxable income." 13 The
although a few race horses were also raised. It does not appear that the farm Commissioner advances no valid grounds in his brief for contesting the Tax
was used by petitioner for entertainment, social activities, or other non-business Court's findings. Certainly, these increases in the taxpayer's net worth were not
purposes. Therefore, it is entitled to deduct expenses and losses in connection taxable increases in net worth, as they were not the result of the receipt by it of
with the operation of said farm. (See 1955 PH Fed. Taxes, Par. 13, 63, citing unreported or unexplained taxable income, but were shown to be merely the
G.C.M. 21103, CB 1939-1, p.164) result of the correction of errors in its entries in its books relating to its
Section 100 of Revenue Regulations No. 2, otherwise known as the indebtednesses to certain creditors, which had been erroneously overstated or
Income Tax Regulations, authorizes farmers to determine their gross listed as outstanding when they had in fact been duly paid. The Tax Court's
income on the basis of inventories. Said regulations provide: action must be affirmed.
"If gross income is ascertained by inventories, no 4. Gain realized from sale of real property (1950). — We likewise sustain as
deduction can be made for livestock or products lost being in accordance with the evidence the Tax Court's reversal of the
during the year, whether purchased for resale, produced Commissioner's assessment on all alleged unreported gain in the sum of
on the farm, as such losses will be reflected in the P11,147.26 in the sale of a certain real property of the taxpayer in 1950. As
inventory by reducing the amount of livestock or found by the Tax Court, the evidence shows that this property was acquired in
products on hand at the close of the year." 1926 for P11,852.74, and was sold in 1950 for P60,000.00, apparently, resulting
in a gain of P48,147.26. 14 The taxpayer reported in its return a gain of
Evidently, petitioner determined its income or losses in the operation of said P37,000.00, or a discrepancy of P11,147.26. 15 It was sufficiently proved from
farm on the basis of inventories. We quote from the memorandum of counsel the taxpayer's books that after acquiring the property, the taxpayer had made
for petitioner: improvements totalling P11,147.26, 16 accounting for the apparent discrepancy
in the reported gain. In other words, this figure added to the original acquisition
cost of P11,852.74 results in a total cost of P23,000.00, and the gain derived income or losses, and the Commissioner having failed to show the contrary,
from the sale of the property for P60,000.00 was correctly reported by the we reiterate our ruling [supra, paragraph 1 (d) and (e)] that we find no
taxpayer at P37,000.00. compelling reason to disturb its findings.
2ND ISSUE: 6. Disallowance of amortization of alleged "contractual rights." — The reasons
for sustaining this disallowance are thus given by the Tax Court:
PETITIONER CONTENTION: that the Commissioner's action to recover
its tax liability should be deemed to have prescribed for failure on the part It appears that the Palawan Manganese Mines, Inc., during a special
of the Commissioner to file a complaint for collection against it in an meeting of its Board of Directors on January 19, 1956, approved a
appropriate civil action, as contradistinguished from the answer filed by the resolution, the pertinent portions of which read as follows:
Commissioner to its petition for review of the questioned assessments in the
"RESOLVED, as it is hereby resolved, that the
case a quo has long been rejected by this Court. T
corporation's current assets composed of ores, fuel, and
COURT: his Court has consistently held that "a judicial action for the oil, materials and supplies, spare parts and canteen
collection of a tax is begun by the filing of a complaint with the proper supplies appearing in the inventory and balance sheet of
court of first instance, or where the assessment is appealed to the Court of the Corporation as of December 31, 1955, with an
Tax Appeals, by filing an answer to the taxpayer's petition for review wherein aggregate value of P97,636.98, contractual rights for the
payment of the tax is prayed for." 17 This is but logical for where the taxpayer operation of various mining claims in Palawan with a
avails of the right to appeal the tax assessment to the Court of Tax Appeals, the value of P100,000.00, its title on various mining claims
said Court is vested with the authority to pronounce judgment as to the in Palawan with a value of P142,408.10 or a total value
taxpayer's liability to the exclusion of any other court. In the present case, of P340,045.02 be, as they are hereby ceded and
regardless of whether the assessments were made on February 24 and 27, transferred to Fernandez Hermanos, Inc., as partial
1956, as claimed by the Commissioner, or on December 27, 1955 as claimed settlement of the indebtedness of the corporation to said
by the taxpayer, the government's right to collect the taxes due has clearly Fernandez Hermanos Inc. in the amount of
not prescribed, as the taxpayer's appeal or petition for review was filed P442,895.23." (Exh. E, p. 17, CTA rec.)
with the Tax Court on May 4, 1960, with the Commissioner filing on May
On March 29, 1956, petitioner's corporation accepted the above
20, 1960 his Answer with a prayer for payment of the taxes due, long
offer of transfer, thus:
before the expiration of the five-year period to effect collection by judicial
action counted from the date of assessment. "WHEREAS, the Palawan Manganese Mines, Inc., due
to its yearly substantial losses has decided to cease
Cases L-24972 and L-24978
operation on January 1, 1956 and in order to satisfy at
These cases refer to the taxpayer's income tax liability for the year 1957. least a part of its indebtedness to the Corporation, it has
Upon examination of its corresponding income tax return, the Commissioner proposed to transfer its current assets in the amount of
assessed it for deficiency income tax in the amount of P38,918.76, computed as NINETY SEVEN THOUSAND SIX HUNDRED
follows: THIRTY SIX PESOS & 98/100 (P97,636.98) as per its
balance sheet as of December 31, 1955, its contractual
Net income per return P29,178.70 rights valued at ONE HUNDRED THOUSAND PESOS
(P100,000.00) and its title over various mining claims
Add: Unallowable deductions: valued at ONE HUNDRED FORTY TWO THOUSAND
(1) Net loss claimed on Ha. Dalupiri 89,547.33 FOUR HUNDRED EIGHT PESOS & 10/100
(2) Amortization of Contractual right claimed as an expense (P142,408.10) or a total evaluation of THREE
under Mines Operations 48,481.62 HUNDRED FORTY THOUSAND FORTY FIVE
Net income per investigation PESOS & 08/100 (P340,045.08) which shall be applied
in partial settlement of its obligation to the Corporation
P167,297.65 in the amount of FOUR HUNDRED FORTY TWO
Tax due thereon 38,818.00 THOUSAND EIGHT HUNDRED EIGHTY FIVE
Less: Amount already assessed PESOS & 23/100 (P442,885.23)," (Exh. E-1, p. 18, CTA
rec.)
5,836.00 Petitioner determined the cost of the mines at P242,408.10 by
Balance P32,982.00 adding the value of the contractual rights (P100,000.00) and the
Add: 1/2% monthly interest from 6-20-59 to 6-20-62 value of its mining claims (P142,408.10).
5,936.76
TOTAL AMOUNT DUE AND COLLECTIBLE Respondent disallowed the deduction on the following grounds:
(1) that the Palawan Manganese Mines, Inc. could not transfer
P38,918.76
18 P242,408.10 worth of assets to petitioner because the balance sheet
of the said corporation for 1955 shows that it had only current as
worth P97,636.96; and
The Tax Court overruled the Commissioner's disallowance of the taxpayer's
losses in the operation of its Hacienda Dalupiri in the sum of P89,547.33 but (2) that the alleged amortization of "contractual rights" is not
sustained the disallowance of the sum of P48,481.62, which allegedly allowed by the Revenue Code.
represented 1/5 of the cost of the "contractual right" over the mines of its
subsidiary, Palawan Manganese Mines, Inc. which the taxpayer had acquired. It The law in point is Section 30(g) (1) (B) of the Revenue Code, before its
found the taxpayer liable for deficiency income tax for the year 1957 in the amendment by Republic Act No. 2698, which provided in part:
amount of P9,696.00, instead of P32,982.00 as originally assessed, and "(g) Depletion of oil and gas wells and mines.:
rendered the following judgment:
"(1) In general. — ... (B) in the case of mines, a
Both parties again appealed from the respective adverse rulings against them in reasonable allowance for depletion thereof not to exceed
the Tax Court's decision. the market value in the mine of the product thereof,
5. Allowance of losses in Hacienda Dalupiri (1957). — The Tax Court cited its which has been mined and sold during the year for which
previous decision overruling the Commissioner's disallowance of losses the return and computation are made. The allowances
suffered by the taxpayer in the operation of its Hacienda Dalupiri, since it was shall be made under rules and regulations to be
convinced that the hacienda was operated for business and not for pleasure. prescribed by the Secretary of Finance: Provided, That
And in this appeal, the Commissioner cites his arguments in his appellant's when the allowances shall equal the capital invested, ...
brief in Case No. L-21557. The Tax Court, in setting aside the Commissioner's no further allowance shall be made."
principal objections, which were directed to the accounting method used by the Assuming, arguendo, that the Palawan Manganese Mines, Inc. had
taxpayer found that: assets worth P242,408.10 which it actually transferred to the
It is true that petitioner followed the cash basis method of reporting petitioner in 1956, the latter cannot just deduct one-fifth (1/5) of said
income and expenses in the operation of the Hacienda Dalupiri and amount from its gross income for the year 1957 because such
used the accrual method with respect to its mine operations. This deduction in the form of depletion charge was not sanctioned by
method of accounting, otherwise known as the hybrid method, Section 30(g) (1) (B) of the Revenue Code, as above-quoted.
followed by petitioner is not without justification. xxx xxx xxx
... A taxpayer may not, ordinarily, combine the cash and The sole basis of petitioner in claiming the amount of P48,481.62
accrual bases. The 1954 Code provisions permit, as a deduction was the memorandum of its mining engineer
however, the use of a hybrid method of accounting, (Exh. 1, pp. 31-32, CTA rec.), who stated that the ore reserves of the
combining a cash and accrual method, under Busuange Mines (Mines transferred by the Palawan Manganese
circumstances and requirements to be set out in Mines, Inc. to the petitioner) would be exhausted in five (5) years,
Regulations to be issued. Also, if a taxpayer is engaged hence, the claim for P48,481.62 or one-fifth (1/5) of the alleged cost
in more than one trade or business he may use a different of the mines corresponding to the year 1957 and every year
method of accounting for each trade or business. And a thereafter for a period of 5 years. The said memorandum merely
taxpayer may report income from a business on accrual showed the estimated ore reserves of the mines and it probable
basis and his personal income on the cash basis.' (See selling price. No evidence whatsoever was presented to show the
Mertens, Law of Federal Income Taxation, Zimet & produced mine and for how much they were sold during the year for
Stanley Revision, Vol. 2, Sec. 12.08, p. 26.) 20 which the return and computation were made. This is necessary in
The Tax Court, having satisfied itself with the adequacy of the taxpayer's order to determine the amount of depletion that can be legally
accounting method and procedure as properly reflecting the taxpayer's deducted from petitioner's gross income. The method employed by
petitioner in making an outright deduction of 1/5 of the cost of the
mines is not authorized under Section 30(g) (1) (B) of the Revenue
Code. Respondent's disallowance of the alleged "contractual rights"
amounting to P48,481.62 must therefore be sustained. 21
PETITIONER INSISTS: that it could use as a method for depletion under the
pertinent provision of the Tax Code its "capital investment," representing the
alleged value of its contractual rights and titles to mining claims in the sum of
P242,408.10 and thus deduct outright one-fifth (1/5) of this "capital
investment" every year. regardless of whether it had actually mined the product
and sold the products.
COURT: The very authorities cited in its brief give the correct concept of
depletion charges that they "allow for the exhaustion of the capital value of the
deposits by production"; thus, "as the cost of the raw materials must be
deducted from the gross income before the net income can be determined, so
the estimated cost of the reserve used up is allowed." 22 The alleged "capital
investment" method invoked by the taxpayer is not a method of depletion,
but the Tax Code provision, prior to its amendment by Section 1, of
Republic Act No. 2698, which took effect on June 18, 1960, expressly
provided that "when the allowances shall equal the capital invested ... no
further allowances shall be made;" in other words, the "capital
investment" was but the limitation of the amount of depletion that could be
claimed. The outright deduction by the taxpayer of 1/5 of the cost of the mines,
as if it were a "straight line" rate of depreciation, was correctly held by the Tax
Court not to be authorized by the Tax Code.

S-ar putea să vă placă și